top of page

Endocarditis


 A 26 year old female presents fever and chills. 

She is febrile and tachycardia, normal BP.

You think you hear a murmur on the exam, and further questioning, she says she regularly uses iv drugs.

What would you consider,  and what are your next steps? 

Infective Endocarditis

Oct 16, 2017 

Author: John L Brusch, MD, FACP; Chief Editor: Michael Stuart Bronze, MD  more...

Practice Essentials

Infective endocarditis (IE) is defined as an infection of the endocardial surface of the heart (see the image below), which may include one or more heart valves, the mural endocardium, or a septal defect. Its intracardiac effects include severe valvular insufficiency, which may lead to intractable congestive heart failure and myocardial abscesses. If left untreated, IE is almost inevitably fatal.

Signs and symptoms

Fever, possibly low-grade and intermittent, is present in 90% of patients with IE. Heart murmurs are heard in approximately 85% of patients.

One or more classic signs of IE are found in as many as 50% of patients. They include the following:

  • Petechiae: Common, but nonspecific, finding

  • Subungual (splinter) hemorrhages: Dark-red, linear lesions in the nail beds

  • Osler nodes: Tender subcutaneous nodules usually found on the distal pads of the digits

  • Janeway lesions: Nontender maculae on the palms and soles

  • Roth spots: Retinal hemorrhages with small, clear centers; rare

Signs of neurologic disease, which occur in as many as 40% of patients, include the following [1] :

  • Embolic stroke with focal neurologic deficits: The most common neurologic sign

  • Intracerebral hemorrhage

  • Multiple microabscesses

Other signs of IE include the following:

  • Splenomegaly

  • Stiff neck

  • Delirium

  • Paralysis, hemiparesis, aphasia

  • Conjunctival hemorrhage

  • Pallor

  • Gallops

  • Rales

  • Cardiac arrhythmia

  • Pericardial rub

  • Pleural friction rub

Subacute native valve endocarditis

The symptoms of early subacute native valve endocarditis (NVE) are usually subtle and nonspecific; they include the following:

  • Low-grade fever: Absent in 3-15% of patients

  • Anorexia

  • Weight loss

  • Influenza-like syndromes

  • Polymyalgia-like syndromes

  • Pleuritic pain

  • Syndromes similar to rheumatic fever, such as fever, dulled sensorium (as in typhoid), headaches

  • Abdominal symptoms, such as right upper quadrant pain, vomiting, postprandial distress, appendicitis-like symptoms

Diagnosis

The Duke diagnostic criteria, developed by Durack and colleagues, are generally used to make a definitive diagnosis of IE. The criteria combine the clinical, microbiologic, pathologic, and echocardiographic characteristics of a specific case [2] :

Major blood culture criteria for IE include the following:

  • Two blood cultures positive for organisms typically found in patients with IE

  • Blood cultures persistently positive for one of these organisms, from cultures drawn more than 12 hours apart

  • Three or more separate blood cultures drawn at least 1 hour apart

Major echocardiographic criteria include the following:

  • Echocardiogram positive for IE, documented by an oscillating intracardiac mass on a valve or on supporting structures, in the path of regurgitant jets, or on implanted material, in the absence of an alternative anatomic explanation

  • Myocardial abscess

  • Development of partial dehiscence of a prosthetic valve

  • New-onset valvular regurgitation

Minor criteria for IE include the following:

  • Predisposing heart condition or intravenous drug use

  • Fever of 38°C (100.4°F) or higher

  • Vascular phenomenon, including major arterial emboli, septic pulmonary infarcts, mycotic aneurysm, intracranial hemorrhage, conjunctival hemorrhage, or Janeway lesions

  • Immunologic phenomenon such as glomerulonephritis, Osler nodes, Roth spots, and rheumatoid factor

  • Positive blood culture results not meeting major criteria or serologic evidence of active infection with an organism consistent with IE

  • Echocardiogram results consistent with IE but not meeting major echocardiographic criteria

A definitive clinical diagnosis can be made based on the following:

  • 2 major criteria

  • 1 major criterion and 3 minor criteria

  • 5 minor criteria

.

Management

Antibiotics remain the mainstay of treatment for IE. Three to five sets of blood cultures should be obtained within 60-90 minutes, followed by the infusion of the appropriate antibiotic regimen. By necessity, the initial antibiotic choice is empiric in nature, determined by clinical history and physical examination findings.

Empiric antibiotic therapy is chosen based on the most likely infecting organisms. Native valve endocarditis (NVE) has often been treated with penicillin G and gentamicin for synergistic coverage of streptococci. Patients with a history of intravenous drug use have been treated with nafcillin and gentamicin to cover for methicillin-sensitive staphylococci. The emergence of methicillin-resistant Staphylococcus aureus (MRSA) and penicillin-resistant streptococci has led to a change in empiric treatment, with liberal substitution of vancomycin in lieu of a penicillin antibiotic.

Background

Infective endocarditis (IE) is defined as an infection of the endocardial surface of the heart, which may include one or more heart valves, the mural endocardium, or a septal defect. Its intracardiac effects include severe valvular insufficiency, which may lead to intractable congestive heart failure and myocardial abscesses. IE also produces a wide variety of systemic signs and symptoms through several mechanisms, including both sterile and infected emboli and various immunological phenomena. [3, 4, 5]

The history of IE can be divided into several eras. In 1674, Lazaire Riviere first described the gross autopsy findings of the disease in his monumental work Opera medica universa. In 1885, William Osler presented the first comprehensive description of endocarditis in English. Lerner and Weinstein presented a thorough discussion of this disease in modern times in their landmark series of articles, “Infective Endocarditis in the Antibiotic Era,” published in 1966 in the New England Journal of Medicine. [6, 7, 8]

IE currently can be described as infective endocarditis in the era of intravascular devices, as infection of intravascular lines has been determined to be the primary risk factor for Staphylococcus aureus bloodstream infections (BSIs). S aureus has become the primary pathogen of endocarditis. [9] As it evolves, IE continues to pose significant clinical challenges. The mortality rate within one year of acquiring infection is almost 30%. Because of lack of funding to conduct well-designed randomized controlled trials, issues such as which individuals would benefit from antibiotic prophylaxis and when an infected native or prosthetic valve should undergo surgery require further research. [10]

IE generally occurs as a consequence of nonbacterial thrombotic endocarditis, which results from turbulence or trauma to the endothelial surface of the heart. A transient bacteremia then seeds the sterile platelet/fibrin thrombus, with IE as the end result. Pathologic effects due to infection can include local tissue destruction and embolic phenomena. In addition, secondary autoimmune effects, such as immune complex glomerulonephritis and vasculitis, can occur. (See Pathophysiology.)

IE remains a diagnostic and therapeutic challenge. Its manifestations may be muted by the indiscriminate use of antimicrobial agents or by underlying conditions in frail and elderly individuals or immunosuppressed persons. (See Diagnosis.)

Effective therapy has become progressively more difficult to achieve because of the proliferation of implanted biomechanical devices and the rise in the number of resistant organisms. Antibiotic prophylaxis has probably had little effect in decreasing the incidence of IE. (See Treatment and Management.)

For other discussions on IE, see Pediatric Bacterial Endocarditis, Infectious Endocarditis, Neurological Sequelae of Infective Endocarditis, and Antibiotic Prophylactic Regimens for Endocarditis.

Types of infective endocarditis

Endocarditis has evolved into several variations, keeping it near the top of the list of diseases that must not be misdiagnosed or overlooked. Endocarditis can be broken down into the following categories:

  • Native valve endocarditis (NVE), acute and subacute

  • Prosthetic valve endocarditis (PVE), [11] early and late

  • Intravenous drug abuse (IVDA) endocarditis

Other terms commonly used to classify types of IE include pacemaker IE and nosocomial IE (NIE).

The classic clinical presentation and clinical course of IE has been characterized as either acute or subacute. Indiscriminate antibiotic usage and an increase in immunosuppressed patients have blurred the distinction between these 2 major types; however, the classification still has clinical merit. [12]

Acute NVE frequently involves normal valves and usually has an aggressive course. It is a rapidly progressive illness in persons who are healthy or debilitated. Virulent organisms, such as S aureus and group B streptococci, are typically the causative agents of this type of endocarditis. Underlying structural valve disease may not be present.

Subacute NVE typically affects only abnormal valves. Its course, even in untreated patients, is usually more indolent than that of the acute form and may extend over many months. Alpha-hemolytic streptococci or enterococci, usually in the setting of underlying structural valve disease, typically are the causative agents of this type of endocarditis.

PVE accounts for 10-20% of cases of IE. Eventually, 5% of mechanical and bioprosthetic valves become infected. Mechanical valves are more likely to be infected within the first 3 months of implantation, and, after 1 year, bioprosthetic valves are more likely to be infected. The valves in the mitral valve position are more susceptible than those in the aortic areas. [11]

Early PVE occurs within 60 days of valve implantation. Traditionally, coagulase-negative staphylococci, gram-negative bacilli, and Candida species have been the common infecting organisms. Late PVE occurs 60 days or more after valve implantation. Staphylococci, alpha-hemolytic streptococci, and enterococci are the common causative organisms. Recent data suggest that S aureus may now be the most common infecting organism in both early and late PVE. [13]

In 75% of cases of IVDA IE, no underlying valvular abnormalities are noted, and 50% of these infections involve the tricuspid valve. [14] S aureus is the most common causative organism.

Analogous to PVE are infections of implantable pacemakers and cardioverter-defibrillators. Usually, these devices are infected within a few months of implantation. Infection of pacemakers includes that of the generator pocket (the most common), infection of the proximal leads, and infection of the portions of the leads in direct contact with the endocardium.

This last category represents true pacemaker IE, is the least common infectious complication of pacemakers (0.5% of implanted pacemakers), and is the most challenging to treat. Of pacemaker infections, 75% are produced by staphylococci, both coagulase-negative and coagulase-positive.

NIE is defined as an infection that manifests 48 hours after the patient is hospitalized or that is associated with a hospital, based on a procedure performed within 4 weeks of clinical disease onset. The term healthcare-associated infective endocarditis (HCIE) is preferable to NIE, since it is inclusive of all sites that deliver patient care, such as hemodialysis centers. The term NIE should be applied to cases of IE acquired in the hospital. An appropriate alternative term would be iatrogenic IE.

Two types of NIE have been described. The right-sided variety affects a valve that has been injured by placement of an intravascular line (eg, Swan-Ganz catheter). Subsequently, the valve is infected by a nosocomial bacteremia. The second type develops in a previously damaged valve and is more likely to occur on the left side. S aureus has been the predominant pathogen of NIE/HCIE since the recent prevalence of intravascular devices. Enterococci are second most commonly isolated pathogens. These usually arise from a genitourinary source.

Evolution of clinical characteristics of infective endocarditis

Since the 1960s, the clinical characteristics of IE have changed significantly. The dramatic “graying” of the disease and the increase in recreational drug use and proliferation of invasive vascular procedures underlie this phenomenon. Varieties of IE that were uncommon in the early antibiotic era have become prominent. Cases of NIE, IVDA IE, and PVE have markedly increased. Valvular infections have entered the era of IE caused by intravascular devices and procedures.

The underlying valvular pathology has also changed. Rheumatic heart disease currently accounts for less than 20% of cases, and 6% of patients with rheumatic heart disease eventually develop IE. Approximately 50% of elderly patients have calcific aortic stenosis as the underlying pathology. Congenital heart disease accounts for 15% of cases, with the bicuspid aortic valve being the most common example.

Other contributing congenital abnormalities include ventricular septal defects, patent ductus arteriosus, and tetralogy of Fallot. Atrial septal defect (secundum variety) is rarely associated with IE. Mitral valve prolapse is the most common predisposing condition found in young adults and is the predisposing condition in 30% of cases of NVE in this age group. IE complicates 5% of cases of asymmetrical septal hypertrophy, usually involving the mitral valve.

Pathophysiology

IE develops most commonly on the mitral valve, closely followed in descending order of frequency by the aortic valve, the combined mitral and aortic valve, the tricuspid valve, and, rarely, the pulmonic valve. Mechanical prosthetic and bioprosthetic valves exhibit equal rates of infection.

All cases of IE develop from a commonly shared process, as follows:

  1. Bacteremia (nosocomial or spontaneous) that delivers the organisms to the surface of the valve

  2. Adherence of the organisms

  3. Eventual invasion of the valvular leaflets

The common denominator for adherence and invasion is nonbacterial thrombotic endocarditis, a sterile fibrin-platelet vegetation. The development of subacute IE depends on a bacterial inoculum sufficient to allow invasion of the preexistent thrombus. This critical mass is the result of bacterial clumping produced by agglutinating antibodies.

In acute IE, the thrombus may be produced by the invading organism (ie, S aureus) or by valvular trauma from intravenous catheters or pacing wires (ie, NIE/HCIE). S aureus can invade the endothelial cells (endotheliosis) and increase the expression of adhesion molecules and of procoagulant activity on the cellular surface. Nonbacterial thrombotic endocarditis may result from stress, renal failure, malnutrition, systemic lupus erythematosus, or neoplasia.

The Venturi effect also contributes to the development and location of nonbacterial thrombotic endocarditis. This principle explains why bacteria and the fibrin-platelet thrombus are deposited on the sides of the low-pressure sink that lies just beyond a narrowing or stenosis.

In patients with mitral insufficiency, bacteria and the fibrin-platelet thrombus are located on the atrial surface of the valve. In patients with aortic insufficiency, they are located on the ventricular side. In these examples, the atria and ventricles are the low-pressure sinks. In the case of a ventricular septal defect, the low-pressure sink is the right ventricle and the thrombus is found on the right side of the defect.

Nonbacterial thrombotic endocarditis may also form on the endocardium of the right ventricle, opposite the orifice that has been damaged by the jet of blood flowing through the defect (ie, the MacCallum patch).

The microorganisms that most commonly produce endocarditis (ie, S aureus; Streptococcus viridans; group A, C, and G streptococci; enterococci) resist the bactericidal action of complement and possess fibronectin receptors for the surface of the fibrin-platelet thrombus. Among the many other characteristics of IE-producing bacteria demonstrated in vitro and in vivo, some features include the following:

  • Increased adherence to aortic valve leaflet disks by enterococci, S viridans,and S aureus

  • Mucoid-producing strains of S aureus

  • Dextran-producing strains of S viridans

  • S viridans and enterococci that possess FimA surface adhesin

  • Platelet aggregation by S aureus and S viridans and resistance of S aureusto platelet microbicidal proteins

The pathogenesis of pacemaker IE is similar. Shortly after implantation, the development of a fibrin-platelet thrombus (similar to the nonbacterial thrombotic endocarditis described above) involves the generator box and conducting leads. After 1 week, the connective tissue proliferates, partially embedding the leads in the wall of the vein and endocardium. This layer may offer partial protection against infection during a bacteremia.

Bacteremia (either spontaneous or due to an invasive procedure) infects the sterile fibrin-platelet vegetation described above. BSIs develop from various extracardiac types of infection, such as pneumonias or pyelonephritis, but most commonly from gingival disease. Of those with high-grade gingivitis, 10% have recurrent transient bacteremias (usually streptococcal species). Most cases of subacute disease are secondary to the bacteremias that develop from the activities of daily living (eg, brushing teeth, bowel movements).

The skin is quite resistant to S aureus infection due in great part to its production of antimicrobial peptides. Soong et al discovered that, in vitro, the secretion of alpha toxin by S aureus allows the organism to successfully penetrate the keratinocyte layer. This could explain the presence of staphylococcal bacteremia in the absence of any gross damage to the epithelial layer. [15]

Bacteremia can result from various invasive procedures, ranging from oral surgery to sclerotherapy of esophageal varices to genitourinary surgeries to various abdominal operations. The potential for invasive procedures to produce a bacteremia varies greatly. Procedures, rates, and organisms are as follows:

  • Endoscopy - Rate of 0-20%; coagulase-negative staphylococci (CoNS), streptococci, diphtheroids

  • Colonoscopy - Rate of 0-20%; Escherichia coli, Bacteroides species

  • Barium enema - Rate of 0-20%; enterococci, aerobic and anaerobic gram-negative rods

  • Dental extractions - Rate of 40-100%; S viridans

  • Transurethral resection of the prostate - Rate of 20-40%; coliforms, enterococci, S aureus

  • Transesophageal echocardiography - Rate of 0-20%; S viridans, anaerobic organisms, streptococci

The incidence of nosocomial bacteremias, mostly associated with intravascular lines, has more than doubled in the last few years. Up to 90% of BSIs caused by these devices are secondary to the placement of various types of central venous catheters. Hickman and Broviac catheters are associated with the lowest rates, presumably because of their Dacron cuffs. Peripherally placed central venous catheters are associated with similar rates.

Intravascular catheters are infected from 1 of the following 4 sources:

  • Infection of the insertion site

  • Infection of the catheter

  • Bacteremia arising from another site

  • Contamination of the infused solution

Bacterial adherence to intravascular catheters depends on the response of the host to the presence of this foreign body, the properties of the organism itself, and the position of the catheter. Within a few days of insertion, a sleeve of fibrin and fibronectin is deposited on the catheter. S aureus adheres to the fibrin component.

S aureus also produces an infection of the endothelial cells (endotheliosis), which is important in producing the continuous bacteremia of S aureus BSIs. Endotheliosis may explain many cases of persistent methicillin-susceptible S aureus (MSSA) and methicillin-resistant S aureus (MRSA) catheter-related BSIs without an identifiable cause.

S aureus catheter-related BSIs occur even after an infected catheter is removed, apparently attributable to specific virulence factors of certain strains of S aureusthat invade the adjacent endothelial cells. At some point, the staphylococci re-enter the bloodstream, resulting in bacteremia. [16]

Four days after placement, the risk of infection markedly increases. Lines positioned in the internal jugular are more prone to infection than those placed in the subclavian vein. Colonization of the intracutaneous tract is the most likely source of short-term catheter-related BSIs. Among lines in place for more than 2 weeks, infection of the hub is the major source of bacteremia. In some cases, the infusion itself may be a reservoir of infection.

Colonization of heart valves by microorganisms is a complex process. Most transient bacteremias are short-lived, are without consequence, and are often not preventable. Bacteria rarely adhere to an endocardial nidus before the microorganisms are removed from the circulation by various host defenses.

Once microorganisms do establish themselves on the surface of the vegetation, the process of platelet aggregation and fibrin deposition accelerate at the site. As the bacteria multiply, they are covered by ever-thickening layers of platelets and thrombin, which protect them from neutrophils and other host defenses. Organisms deep in the vegetation hibernate because of the paucity of available nutrients and are therefore less susceptible to bactericidal antimicrobials that interfere with bacterial cell wall synthesis.

Complications of subacute endocarditis result from embolization, slowly progressive valvular destruction, and various immunological mechanisms. The pathological picture of subacute IE is marked by valvular vegetations in which bacteria colonies are present both on and below the surface.

The cellular reaction in SBE is primarily that of mononuclear cells and lymphocytes, with few polymorphonuclear cells. The surface of the valve beneath the vegetation shows few organisms. Proliferation of capillaries and fibroblasts is marked. Areas of healing are scattered among areas of destruction. Over time, the healing process falls behind, and valvular insufficiency develops secondary to perforation of the cusps and damage to the chordae tendineae. Compared with acute disease, little extension of the infectious process occurs beyond the valvular leaflets.

Levels of agglutinating and complement-fixing bactericidal antibodies and cryoglobulins are markedly increased in patients with subacute endocarditis. Many of the extracardiac manifestations of this form of the disease are due to circulating immune complexes. Among these include glomerulonephritis, peripheral manifestations (eg, Osler nodes, Roth spots, subungual hemorrhages), and, possibly, various musculoskeletal abnormalities. Janeway lesions usually arise from infected microemboli.

The microscopic appearance of acute bacterial endocarditis differs markedly from that of subacute disease. Vegetations that contain no fibroblasts develop rapidly, with no evidence of repair. Large amounts of both polymorphonuclear leukocytes and organisms are present in an ever-expanding area of necrosis. This process rapidly produces spontaneous rupture of the leaflets, of the papillary muscles, and of the chordae tendineae.

The complications of acute bacterial endocarditis result from intracardiac disease and metastatic infection produced by suppurative emboli. Because of their shortened course, immunological phenomena are not a part of acute IE.

Etiology

The different types of IE have varying causes and involve different pathogens.

Native valve endocarditis

The following are the main underlying causes of NVE:

  • Rheumatic valvular disease (30% of NVE) - Primarily involves the mitral valve followed by the aortic valve

  • Congenital heart disease (15% of NVE) - Underlying etiologies include a patent ductus arteriosus, ventricular septal defect, tetralogy of Fallot, or any native or surgical high-flow lesion.

  • Mitral valve prolapse with an associated murmur (20% of NVE)

  • Degenerative heart disease - Including calcific aortic stenosis due to a bicuspid valve, Marfan syndrome, or syphilitic disease

Approximately 70% of infections in NVE are caused by Streptococcus species, including S viridans, Streptococcus bovis, and enterococci. Staphylococcusspecies cause 25% of cases and generally demonstrate a more aggressive acute course (see the images below).

Prosthetic valve endocarditis

Early PVE, which presents shortly after surgery, has a different bacteriology and prognosis than late PVE, which presents in a subacute fashion similar to NVE.

Infection associated with aortic valve prostheses is particularly associated with local abscess and fistula formation, and valvular dehiscence. This may lead to shock, heart failure, heart block, shunting of blood to the right atrium, pericardial tamponade, and peripheral emboli to the central nervous system and elsewhere.

Early PVE may be caused by a variety of pathogens, including S aureus and S epidermidis. These nosocomially acquired organisms are often methicillin-resistant (eg, MRSA). [17] Late disease is most commonly caused by streptococci. Overall, CoNS are the most frequent cause of PVE (30%).

S aureus causes 17% of early PVE and 12% of late PVE. Corynebacterium,nonenterococcal streptococci, fungi (eg, C albicans, Candida stellatoidea, Aspergillus species), Legionella, and the HACEK (ie, Haemophilus aphrophilus, Actinobacillus actinomycetemcomitans, Cardiobacterium hominis, Eikenella corrodens, Kingella kingae) organisms cause the remaining cases.

IVDA infective endocarditis

Diagnosis of endocarditis in IV drug users can be difficult and requires a high index of suspicion. Two thirds of patients have no previous history of heart disease or murmur on admission. A murmur may be absent in those with tricuspid disease, owing to the relatively small pressure gradient across this valve. Pulmonary manifestations may be prominent in patients with tricuspid infection: one third have pleuritic chest pain, and three quarters demonstrate chest radiographic abnormalities.

S aureus is the most common (< 50% of cases) etiologic organism in patients with IVDA IE. MRSA accounts for an increasing portion of S aureus infections and has been associated with previous hospitalizations, long-term addiction, and nonprescribed antibiotic use. Groups A, C, and G streptococci and enterococci are also recovered from patients with IVDA IE.

Currently, gram-negative organisms are involved infrequently. P aeruginosa [18]and the HACEK family are the most common examples.

Nosocomial/healthcare-associated infective endocarditis

Endocarditis may be associated with new therapeutic modalities involving intravascular devices such as central or peripheral intravenous catheters, rhythm control devices such as pacemakers and defibrillators, hemodialysis shunts and catheters, and chemotherapeutic and hyperalimentation lines. [19, 20]These patients tend to have significant comorbidities, more advanced age, and predominant infection with S aureus. The mortality rate is high in this group.

The organisms that cause NIE/HCIE obviously are related to the type of underlying bacteremia. The gram-positive cocci (ie, S aureus, CoNS, enterococci, nonenterococcal streptococci) are the most common pathogens.

Fungal endocarditis

Fungal endocarditis is found in intravenous drug users and intensive care unit patients who receive broad-spectrum antibiotics. [21] Blood cultures are often negative, and diagnosis frequently is made after microscopic examination of large emboli.

Clinical features associated with different pathogens

Different causative organisms tend to give rise to varying clinical manifestations of IE, as shown in the Table below.

Table 1. Clinical Features of Infective Endocarditis According to Causative Organism (Open Table in a new window)

Causative Organism(s)Clinical Features of IE

Staphylococcus aureus

  • Overall, S aureus infection is the most common cause of IE, including PVE, acute IE, and IVDA IE.

  • Approximately 35-60.5% of staphylococcal bacteremias are complicated by IE.

  • More than half the cases are not associated with underlying valvular disease.

  • The mortality rate of S aureus IE is 40-50%.

  • S aureus infection is the second most common cause of nosocomial BSIs, second only to CoNS infection.

  • The incidence of MRSA infections, both the hospital- and community-acquired varieties, has dramatically increased (50% of isolates). Sixty percent of individuals are intermittent carriers of MRSA or MSSA .

  • The primary risk factor for S aureus BSI is the presence of intravascular lines. Other risk factors include cancer, diabetes, corticosteroid use, IVDA, alcoholism, and renal failure.

  • The realization that approximately 50% of hospital- and community-acquired staphylococcal bacteremias arise from infected vascular catheters has led to the reclassification of staphylococcal BSIs. BSIs are acquired not only in the hospital but also in any type of health care facility (eg, nursing home, dialysis center).

  • Of S aureus bacteremia cases in the United States, 7.8% (200,000) per year are associated with intravascular catheters.

Streptococcus viridans

  • This organism accounts for approximately 50-60% of cases of subacute disease.

  • Most clinical signs and symptoms are mediated immunologically.

Streptococcus intermedius group

  • These infections may be acute or subacute.

  • S intermedius infection accounts for 15% of streptococcal IE cases.

  • Members of the S intermedius group, especially S anginosus, are unique among the streptococci in that they can actively invade tissue and form abscesses, often in the CNS.

Abiotrophia

  • Approximately 5% of subacute cases of IE are due to infection with Abiotrophia species.

  • They require metabolically active forms of vitamin B-6 for growth.

  • This type of IE is associated with large vegetations that lead to embolization and a high rate of posttreatment relapse.

Group D streptococci

  • Most cases are subacute.

  • The source is the gastrointestinal or genitourinary tract.

  • It is the third most common cause of IE.

  • They pose major resistance problems for antibiotics.

Nonenterococcal group D

  • The clinical course is subacute.

  • Infection often reflects underlying abnormalities of the large bowel (eg, ulcerative colitis, polyps, cancer).

  • The organisms are sensitive to penicillin.

Group B streptococci

  • Acute disease develops in pregnant patients and older patients with underlying diseases (eg, cancer, diabetes, alcoholism).

  • The mortality rate is 40%.

  • Complications include metastatic infection, arterial thrombi, and congestive heart failure.

  • It often requires valve replacement for cure.

Group A, C, and G streptococci

  • Acute disease resembles that of S aureus IE (30-70% mortality rate), with suppurative complications.

  • Group A organisms respond to penicillin alone.

  • Group C and G organisms require a combination of synergistic antibiotics (as with enterococci).

Coagulase-negative S aureus

  • This causes subacute disease.

  • It behaves similarly to S viridans infection.

  • It accounts for approximately 30% of PVE cases and less than 5% of NVE cases. [22]

Staphylococcus lugdunensis

  • Staphylococcus lugdunensis is another coagulase-negative Staphylococcus species but is extremely aggressive compared to coagulase-positive S aureus. S lugdunensis frequently causes IE. [23]

Pseudomonas aeruginosa

  • This is usually acute, except when it involves the right side of the heart in IVDA IE.

  • Surgery is commonly required for cure.

HACEK (ie, Haemophilus aphrophilus, Actinobacillus actinomycetemcomitans, Cardiobacterium hominis, Eikenella corrodens, Kingella kingae)

  • These organisms usually cause subacute disease.

  • They account for approximately 5% of IE cases.

  • They are the most common gram-negative organisms isolated from patients with IE.

  • Complications may include massive arterial emboli and congestive heart failure.

  • Cure requires ampicillin, gentamicin, and surgery.

Fungal

  • These usually cause subacute disease.

  • The most common organism of both fungal NVE and fungal PVE is Candida albicans.

  • Fungal IVDA IE is usually caused by Candida parapsilosisor Candida tropicalis.

  • Aspergillus species are observed in fungal PVE and NIE.

Bartonella

  • The most commonly involved species is Bartonella quintana.

  • IE typically develops in homeless males who have extremely substandard hygiene. Bartonella must be considered in cases of culture-negative endocarditis among homeless individuals.

Multiple pathogens (polymicrobial)

  • Pseudomonas and enterococci are the most common combination of organisms.

  • It is observed in cases of IVDA IE

  • The cardiac surgery mortality rate is twice that associated with single-agent IE. [24]

Risk factors

The most significant risk factor for IE is residual valvular damage caused by a previous attack of endocarditis. [25, 19]

Many possible risk factors for the development of pacemaker IE have been described, including diabetes mellitus, age, and use of anticoagulants and corticosteroids. The evidence for these is conflicting. The major risk factor is probably surgical intervention to any part of the pacemaker system, especially elective battery replacements. The rate of infection associated with battery replacements is approximately 5 times that of the initial implantation (6.5% vs 1.4%).

Other significant risk factors for pacemaker IE include the development of a postoperative hematoma, the inexperience of the surgeon, and a preceding temporary transvenous pacing.

Epidemiology

In the United States, the 2009 incidence of IE was approximately 12.7 cases per 100,000 persons per year. [26] The age-adjusted hospital admission rate has increased 2.4% annually from 1998-2009. This rate has risen significantly from that of the previous 50 years (2-4 cases per 100,000 persons per year). [27] The incidence of IE in other countries is similar to that in the United States. From 1998-2009, the proportion of patients with intracardiac devices increased from 13.3% to 18.9%, while the proportion of cases with a background of HIV infection or HIV drug abuse fell.

Between 1998 and 2009, the mean age of patients has risen from 58.6 to 60.8 years. [26, 28] Currently, more than 50% of patients are older than 50 years. [19]Mendiratta et al, in their retrospective study of hospital discharges from 1993-2003 of patients aged 65 years and older with a primary or secondary diagnosis of IE, found that hospitalizations for IE increased 26%, from 3.19 per 10,000 elderly patients in 1993 to 3.95 per 10,000 in 2003. [29] This increase in age has continued, with the mean age of patients in 2009 at 60.8 years. [26]

IE is 3 times as common in males as in females. It has no racial predilection.

Prognosis

Prognosis largely depends on whether or not complications develop. If left untreated, IE is generally fatal. Early detection and appropriate treatment of this uncommon disease can be lifesaving. The overall mortality rate has remained stable at 14.5%. [26]

Cure rates for appropriately managed (including both medical and surgical therapies) NVE are as follows:

  • For S viridans and S bovis infection, the rate is 98%.

  • For enterococci and S aureus infection in individuals who abuse intravenous drugs, the rate is 90%.

  • For community-acquired S aureus infection in individuals who do not abuse intravenous drugs, the rate is 60-70%.

  • For infection with aerobic gram-negative organisms, the rate is 40-60%.

  • For infection with fungal organisms, the rate is lower than 50%.

For PVE, the cure rates are as follows:

  • Rates are 10-15% lower for each of the above categories, for both early and late PVE.

  • Surgery is required far more frequently.

  • Approximately 60% of early CoNS PVE cases and 70% of late CoNS PVE cases are curable.

Anecdotal reports describe the resolution of right-sided valvular infection caused by S aureus infection in individuals who abuse intravenous drugs after just a few days of oral antibiotics.

The role of early valvular surgery in reducing mortality among patients with IE has become somewhat clearer. Challenges to resolving this question include the necessity of performing multicentered studies with an apparent difficulty of ensuring that the patients' preoperative assessments and surgical approaches are comparable. The largest study to date indicates that in cases of IE complicated by heart failure, valvular surgery reduces the 1-year mortality rate.[30] More recent studies document that early surgery in patients, especially those with large vegetations, significantly reduces the risk of death from any cause that from embolic events. [31, 32]

Mortality rates in NVE range from 16-27%. Mortality rates in patients with PVE are higher. More than 50% of these infections occur within 2 months after surgery. The fatality rate of pacemaker IE ranges up to 34%. [33]

Increased mortality rates are associated with increased age, [34] infection involving the aortic valve, development of congestive heart failure, central nervous system (CNS) complications, and underlying disease such as diabetes mellitus. Catastrophic neurological events of all types due to IE are highly predictive of morbidity and mortality. [35]

Mortality rates also vary with the infecting organism. Acute endocarditis due to S aureus is associated with a high mortality rate (30-40%), except when it is associated with IV drug use. [13, 36] Endocarditis due to streptococci has a mortality rate of approximately 10%.

Patient Education

Surveys indicate that an appallingly small number of patients who are at risk for developing IE have an understanding of antibiotic and nonpharmacologic (ie, appropriate oral hygiene) principles. Drug rehabilitation for patients who use IV drugs is critical.

The United Kingdom’s National Institute for Health and Clinical Excellence (NICE) addresses patient education in its 2008 guideline on prophylaxis against IE in adults and children undergoing interventional procedures. The NICE’s guideline recommends that health care professionals teach patients about the symptoms of IE and the risks of nonmedical invasive procedures such as body piercing and tattooing, explain the benefits and risks of antibiotic prophylaxis and the reasons that it is no longer routine, and emphasize the need to maintain good oral health. [37]

For patient education information, see the Heart Center, as well as Tetralogy of Fallot.

History

In patients with infective endocarditis (IE), the present illness history is highly variable. Symptoms commonly are vague, emphasizing constitutional complaints, or complaints may focus on primary cardiac effects or secondary embolic phenomena. Fever and chills are the most common symptoms; anorexia, weight loss, malaise, headache, myalgias, night sweats, shortness of breath, cough, or joint pains are common complaints as well.

Primary cardiac disease may present with signs of congestive heart failure due to valvular insufficiency. Secondary phenomena could include focal neurologic complaints due to an embolic stroke or back pain associated with vertebral osteomyelitis. As many as 20% of cases present with focal neurologic complaints and stroke syndromes.

Dyspnea, cough, and chest pain are common complaints of intravenous drug users. This is likely related to the predominance of tricuspid valve endocarditis in this group and secondary embolic showering of the pulmonary vasculature.

A key concern is the distinction between subacute and acute IE. The diagnosis of subacute IE is suggested by a history of an indolent process characterized by fever, fatigue, anorexia, back pain, and weight loss. Less common developments include a cerebrovascular accident or congestive heart failure.

The patient should be questioned about invasive procedures and recreational drug use that may be causing the bacteremia. Most subacute disease caused by S viridans infection is related to dental disease. Most cases are not caused by dental procedures but by transient bacteremias secondary to gingivitis. In 85% of patients, symptoms of endocarditis appear within 2 weeks of dental or other procedures.

The interval between the onset of disease and diagnosis averages approximately 6 weeks. The fact that less than 50% of patients have previously diagnosed underlying valvular disease significantly limits the effectiveness of antibiotic prophylaxis.

Acute IE is a much more aggressive disease. The patient notices an acute onset of high-grade fevers and chills and a rapid onset of congestive heart failure. Again, a history of antecedent procedures or illicit drug use must be investigated.

The distinction between these 2 polar types of IE has become less clear. Intermittent use of antibiotics aimed at treating misdiagnosed endocarditis can suppress bacterial growth within the valvular thrombus, giving rise to the state of muted IE. This is often the case in nosocomial infective endocarditis (NIE; also referred to as healthcare-associated IE [HCIE]), which commonly manifests with elements of a sepsis syndrome (ie, hypotension, metabolic acidosis fever, leukocytosis, and multiple organ failure).

The source of the bacteremia may be an infection in another organ (eg, pneumonia, pyelonephritis) or in a central venous catheter. Most often, these patients are in the intensive care unit. Approximately 45% of cases of NIE/HCIE occur in patients with prosthetic valves. Muted IE due to S aureus infection may resemble IE that results from S viridans infection.

Subacute native valve endocarditis

The symptoms of early subacute native valve endocarditis (NVE) are usually subtle and nonspecific. They include low-grade fever (absent in 3-15% of patients), anorexia, weight loss, influenzalike syndromes, polymyalgia-like syndromes, pleuritic pain, syndromes similar to rheumatic fever (eg, fever, dulled sensorium as in typhoid, headaches), and abdominal symptoms (eg, right upper quadrant pain, vomiting, postprandial distress, appendicitis-like symptoms).

When appropriate therapy is delayed for weeks or months, additional clinical features, embolic or immunological in origin, develop.

Signs and symptoms secondary to emboli include acute meningitis with sterile spinal fluid, hemiplegia in the distribution of the middle cerebral artery, regional infarcts that cause painless hematuria, infarction of the kidney or spleen, unilateral blindness caused by occlusion of a retinal artery, and myocardial infarction arising from embolization of a coronary artery.

The emboli of right-sided IE commonly produce pulmonary infarcts. The rate of embolization is related to the organism, the size of the vegetation and its rate of growth or resolution, and its location.

The vegetations of S aureus, Haemophilus influenzae, H parainfluenzae, and the fungi are much more likely to embolize than those of S viridans. Those larger than 10 mm in diameter and mobile or prolapsing have a high rate of embolization. A vegetation that grows during therapy is associated with a significant increase in the risk of embolization but with the persistence of bacteremia.

Clinically separating the importance of the absolute size and the rate of change in the size of the vegetation from the causative organism is difficult. The vegetations of the mitral valve are much more likely to embolize than those in any other location. The risk of embolization markedly decreases after 1 week of appropriate antibiotic therapy.

The deposition of circulating immune complexes in the kidney may produce interstitial nephritis or proliferative glomerulonephritis, with renal failure progressing to the point of uremia at the time of the patient’s presentation. Similarly, various musculoskeletal symptoms (44% of patients) arise from immunologically mediated synovitis.

Osler nodes and Roth spots arise from immune-mediated vasculitis. Patients may experience palpitations, ie, the symptoms of an immune-mediated myocarditis.

The origin of lumbosacral back pain in patients with subacute IE (15%) is unclear but probably results from the deposition of immune complexes in the disk space. However, antibiotic therapy rapidly abolishes these symptoms. In 50% of patients with cerebral emboli, this event is the first manifestation of IE and is associated with a 2- to 4-times higher mortality rate. Stroke in younger people should always raise the possibility of underlying IE.

Bacteria-free infective endocarditis

Rarely observed today, the bacteria-free state of IE is one in which patients have multiple negative blood culture results in the presence of severe congestive heart failure, renal failure, multiple sterile emboli, massive splenomegaly, severe anemia, brown facial pigmentation, bilateral thigh pain, and massive leg edema. These patients are usually afebrile. This process appears to indicate prolonged and unchecked stimulation of the immune system.

Acute infective endocarditis

The clinical symptoms of acute IE result from either embolic or intracardiac suppurative complications. The onset of illness is abrupt, with rapidly progressive destruction of the infected valve (see the images below). The valvular leaflets are quickly destroyed by bacteria that multiply rapidly within the ever-growing friable vegetations. Complications develop within a week. These include the dyspnea and fatigue of severe congestive heart failure and a wide spectrum of neuropsychiatric complications resulting from CNS involvement.

Intravenous-drug-abuse infective endocarditis

Patients with right-sided intravenous drug abuse (IVDA) IE (53% of cases) frequently present with pleuropulmonary (pneumonia and/or empyema) manifestations. Symptoms due to metastatic infection develop early in a disease course caused by S aureus. Right-sided disease is associated with a low rate of congestive heart failure and valvular perforation.

Infection with P aeruginosa has a high rate of neurological involvement, with 2 distinctive features: (1) mycotic aneurysms with a higher-than-average rate of rupture and (2) panophthalmitis (10% of patients). The course of infection with P aeruginosa is much slower than that of S aureus.

The course of left-sided IVDA IE is similar to that of non-IVDA disease.

Approximately 5-8% of febrile individuals who abuse intravenous drugs have underlying IE. Many users of illicit drugs may lose their fever within a few hours of hospitalization. This phenomenon, termed cotton wool fever, is probably caused by the presence of adulterants contained within the injected drugs.

Prosthetic valve endocarditis

Clinical features of prosthetic valve endocarditis (PVE) closely resemble those of NVE. Early PVE is defined as infection occurring within 60 days of valve implantation; late PVE occurs after this period. For valvular infection with coagulase-negative staphylococci (CoNS), this division should be extended to 12 months.

Congestive heart failure occurs earlier and is more severe in persons with PVE. The patient may present with symptoms of myocarditis or pericarditis. The rate of embolic stroke is high in the first 3 days of PVE.

Pacemaker infective endocarditis

The clinical presentation in a person with a pacemaker infection and pacemaker IE depends on several factors, including the site of infection (eg, generator pocket vs intravascular leads or epicardial leads), the type of organism, and the origin of the infection (eg, pocket erosion, localized infection of the generator pocket, bacteremia from a remote site). The risk of developing IE is directly associated with the complexity of the permanent pacemaker. The risk associated with internal cardiac defibrillators was almost twice that of single-chamber pacemakers. Mortality also was greater in the former group. [38]

Half of cases of right-sided IE associated with a cardiac device are due to coagulase-negative staphylococci. [39]

Early infections, within a few months of implantation, manifest as acute or subacute infections of the pulse-generator pocket. Bacteremia may be present even in the absence of clinical signs and symptoms. Fever is the most common finding and may be the only finding in approximately 33% of patients.

Late infections of the pocket may be due to erosion of the overlying skin without systemic involvement. Such erosions always indicate infection of the underlying device.

The most significant late infections involve the transvenous or epicardial leads. With epicardial infection, signs and symptoms of pericarditis or mediastinitis may be present along with bacteremia. Infection of the transvenous electrode produces signs and symptoms of right-sided endocarditis. Those that occur early after implantation (33% of cases) show prominent systemic signs of infection, often with obvious localization to the pacemaker pocket.

Late infections have much more subtle manifestations. They may occur up to several years after implantation or reimplantation.

Fever is almost universal in persons with pacemaker IE. Signs of right-sided endocarditis (ie, pneumonia, septic emboli) are observed in up to 50% of patients.

Nosocomial infective endocarditis

NIE commonly manifests with elements of a sepsis syndrome (ie, hypotension, metabolic acidosis fever, leukocytosis, and multiple organ failure). The source of bacteremia may develop from an infection in another organ (eg, pneumonia, pyelonephritis) or from a central venous catheter. Most often, these patients are in the intensive care unit.

The aging of the population is associated with an increased incidence of staphylococcal healthcare-associated endocarditis, in addition to an increased mortality rate associated with the disease. [40]

Approximately 45% of cases of NIE/HCIE occur in patients with prosthetic valves.

IE associated with dialysis catheters has a fairly high rate of MRSA involvement. Eighty percent of cases involve the mitral valve. [41]

Physical Examination

General findings

Fever, possibly low-grade and intermittent, is present in 90% of patients.

The AHA (endorsed by the Infectious Diseases Society of America [IDSA]) 2010 guideline update on cardiovascular implantable electronic device (CIED) infections and their management recommends that patients with CIED who develop unexplained fever or bloodstream infection should seek evaluation for CIED infection by cardiologists or infectious disease specialists. [42]

Heart murmurs are heard in approximately 85% of patients. Change in the characteristics of a previously noted murmur occurs in 10% of these patients and increases the likelihood of secondary congestive heart failure.

One or more classic signs of IE are found in as many as 50% of patients. They include the following:

  • Petechiae - Common but nonspecific finding (see the image below)

  • Subungual (splinter) hemorrhages - Dark red linear lesions in the nailbeds

  • Osler nodes - Tender subcutaneous nodules usually found on the distal pads of the digits

  • Janeway lesions - Nontender maculae on the palms and soles

  • Roth spots - Retinal hemorrhages with small, clear centers; rare and observed in only 5% of patients

Signs of neurologic disease occur in as many as 40% of patients. Embolic stroke with focal neurologic deficits is the most common etiology. Other etiologies include intracerebral hemorrhage and multiple microabscesses. [1]

Signs of systemic septic emboli are due to left heart disease and are more commonly associated with mitral valve vegetations. Multiple embolic pulmonary infections or infarctions are due to right heart disease.

Signs of congestive heart failure, such as distended neck veins, frequently are due to acute left-sided valvular insufficiency.

Splenomegaly may be present.

Other signs include the following:

  • Stiff neck

  • Delirium

  • Paralysis, hemiparesis, aphasia

  • Conjunctival hemorrhage

  • Pallor

  • Gallops

  • Rales

  • Cardiac arrhythmia

  • Pericardial rub

  • Pleural friction rub

Subacute infective endocarditis

Approximately 3-15% of patients with subacute IE (primarily elderly and chronically ill individuals) have normal or subnormal temperatures. The vast majority of patients have detectable heart murmurs. The presence of a murmur is so common (99% of cases) that its absence should cause clinicians to reconsider the diagnosis of IE. The major exception is right-sided IE, in which only one third of patients have a detectable murmur.

Because many of these murmurs are hemodynamically insignificant and have been present for years, their role in the patient’s illness may be underestimated. The saying "a changing murmur is extremely helpful in diagnosing subacute IE" is a myth. Only 15% do so early in the course of infection.

The peripheral lesions of subacute IE are observed in only approximately 20% of patients, compared with 85% in the preantibiotic era. Currently, the most common of these is petechiae. They may occur on the palpebral conjunctivae, the dorsa of the hands and feet, the anterior chest and abdominal walls, the oral mucosa, and the soft palate.

Subungual hemorrhages (ie, splinter hemorrhages) are linear and red. They are usually caused by workplace trauma to the hands and feet rather than by valvular infection. Hemorrhages that do not extend for the entire length of the nail are more likely the result of infection rather than trauma.

Osler nodes are smallish tender nodules that range from red to purple and are located primarily in the pulp spaces of the terminal phalanges of the fingers and toes, soles of the feet, and the thenar and hypothenar eminences of the hands. Their appearance is often preceded by neuropathic pain. They last from hours to several days. They remain tender for a maximum of 2 days. The underlying mechanism is probably the circulating immunocomplexes of subacute IE. They have been described in various noninfectious vasculitides.

Clubbing of fingers and toes was found almost universally, but it is now observed in less than 10% of patients. It primarily occurs in those patients who have an extended course of untreated IE.

The arthritis associated with subacute IE is asymmetrical and is limited to 1-3 joints. Clinically, it resembles the joint changes found in patients with rheumatoid arthritis, Reiter syndrome, or Lyme disease. The fluid is usually sterile.

Splenomegaly is observed more commonly in patients with long-standing subacute disease. It may persist long after successful therapy.

Roth spots are retinal hemorrhages with pale centers. The Litten sign represents cotton-wool exudates.

Acute infective endocarditis

In approximately one third of patients with acute IE, murmurs are absent. The most common type is an aortic regurgitation murmur. Because of the suddenness of onset, the left ventricle does not have a chance to dilate. In this situation, the classic finding of increased pulse pressure in significant valvular insufficiency is absent.

Fever is always present, and it usually is high.

Janeway lesions are irregular erythematosus and painless macules (1-4 mm in diameter). They most often are located on the thenar and hypothenar eminences of the hands and feet. They usually represent an infectious vasculitis of acute IE resulting from S aureus infection.

Acute septic monoarticular arthritis in patients with acute IE most often is caused by S aureus infection.

Purulent meningitis may be observed in patients with acute IE, compared with the aseptic type observed in patients with subacute disease. Other neurological findings are similar to those observed in patients with subacute disease. [43, 44,45, 46, 47, 48, 49, 50, 51, 52, 53, 54, 55, 56, 57, 58]

Complications

The following are potential complications of IE:

  • Myocardial infarction, pericarditis, cardiac arrhythmia

  • Cardiac valvular insufficiency

  • Congestive heart failure

  • Sinus of Valsalva aneurysm

  • Aortic root or myocardial abscesses

  • Arterial emboli, infarcts, mycotic aneurysms

  • Arthritis, myositis

  • Glomerulonephritis, acute renal failure

  • Stroke syndromes

  • Mesenteric or splenic abscess or infarct [59]

Congestive heart failure due to aortic valve insufficiency is the most common intracardiac complication of subacute endocarditis. It develops after months of untreated disease but may occur a full year following microbiological cure.

The complication of arterial embolization is second in frequency to congestive heart failure for both subacute and acute IE. The frequency of this complication has decreased, from 80% in the preantibiotic era to 15-35% today. The emboli are usually sterile because of the minimally invasive nature of the causative organisms (eg, S viridans).

The persons most at risk are younger (20-40 y), have mitral or aortic valve (native or prosthetic) involvement, and are infected with certain organisms such as Candida or Aspergillus species, S aureus, Haemophilus parainfluenzae,group B streptococci, and nutritionally variant streptococci.

The prevalence of embolization appears to be the same for both types of disease. The most common areas of deposition include the coronary arteries, kidneys, brain, and spleen. Infarction at the site of embolization is common; abscess formation is not. Cerebral emboli occur in 33% of patients. The middle cerebral artery is involved most often.

Other neurological embolic damage includes cranial nerve palsies, cerebritis, and mycotic aneurysms caused by weakening of the vessel walls and produced by embolization to the vasa vasorum. Mycotic aneurysms may occur in the abdominal aorta and the splenic, coronary, and pulmonary arteries.

In acute IE, the frequency of aneurysms and other suppurative intracardiac complications is high. In addition to valvular insufficiency, other intracardiac complications of acute IE include (1) aortocardiac and other fistulas, (2) aneurysms of the sinus of Valsalva, (3) intraventricular abscesses, (4) ring abscesses, (5) myocardial abscesses, (6) mycotic aneurysms, (7) septic coronary arterial emboli, and (8) pericarditis.

In patients with acute disease, especially disease caused by S aureus infection,emboli almost inevitably lead to abscesses in the areas where they are deposited. Multiple abscesses can occur in almost every organ, including the kidneys, heart, and brain. Mycotic aneurysms may occur in almost any artery. Paradoxically, they are less common in patients with acute IE. [60, 61]

It appears that older patients have higher rates of myocardial infarction and death but lower rates of neurological complications. [26]

Diagnostic Considerations

Definitive diagnosis of infective endocarditis (IE) is generally made by using the Duke criteria.

Duke diagnostic criteria

Durack and colleagues developed diagnostic criteria that combine the clinical, microbiological, pathological, and echocardiographic characteristics of a specific case. [2]

Major blood culture criteria include the following:

  • Two blood cultures positive for organisms typically found in patients with IE (ie, S viridans, Streptococcus bovis, a HACEK group organism, community-acquired S aureus, or enterococci in the absence of a primary focus)

  • Blood cultures persistently positive for one of the above organisms from cultures drawn more than 12 hours apart

  • Three or more separate blood cultures drawn at least 1 hour apart

Major echocardiographic criteria include the following:

  • Echocardiogram positive for IE, documented by an oscillating intracardiac mass on a valve or on supporting structures, in the path of regurgitant jets, or on implanted material in the absence of an alternative anatomical explanation

  • Myocardial abscess

  • Development of partial dehiscence of a prosthetic valve

  • New-onset valvular regurgitation

Minor criteria include the following:

  • Predisposing heart condition or intravenous drug use

  • Fever of 38°C (100.4°F) or higher

  • Vascular phenomenon, including major arterial emboli, septic pulmonary infarcts, mycotic aneurysm, intracranial hemorrhage, conjunctival hemorrhage, or Janeway lesions

  • Immunological phenomenon such as glomerulonephritis, Osler nodes, Roth spots, and rheumatoid factor

  • Positive blood culture results not meeting major criteria or serologic evidence of active infection with an organism consistent with IE (eg,Brucella, C burnetii [ie, Q fever], Legionella)

  • Echocardiogram results consistent with IE but not meeting major echocardiographic criteria

Definitive pathological diagnosis is established by demonstrating microorganisms, by culture or histology, in vegetations removed by surgery, embolectomy, or drainage of an intracardiac abscess. Alternatively, a definitive clinical diagnosis is made based on the presence of 2 major criteria, 1 major criterion and 3 minor criteria, or by 5 minor criteria.

A diagnosis of possible IE is made when findings consistent with IE fall short of the criteria for definite IE but do not meet the criteria for rejection.

Rejection criteria for the diagnosis of IE are as follows:

  • The presence of a firm alternative diagnosis of the manifestations of endocarditis

  • Resolution of manifestations of endocarditis after 4 or fewer days of antimicrobial therapy

  • No pathologic evidence of IE at surgery or autopsy after 4 or fewer days of antimicrobial therapy

These criteria may, at times, overdiagnose IE and may not be as applicable in patients with subacute disease.

Other problems to be considered include the following:

  • Thrombotic nonbacterial endocarditis

  • Vasculitis

  • Temporal arteritis

  • Marantic endocarditis

  • Connective tissue disease

  • Fever of unknown origin (FUO)

  • Intra-abdominal infections

  • Septic pulmonary infarction

  • Tricuspid regurgitation

Differential Diagnoses

  • Antiphospholipid Syndrome

  • Atrial Myxoma

  • Infective Endocarditis

  • Lyme Disease

  • Physical Medicine and Rehabilitation for Systemic Lupus Erythematosus

  • Polymyalgia Rheumatica

  • Primary Cardiac Neoplasms

  • Reactive Arthritis

Approach Considerations

The criterion standard test for diagnosing infective endocarditis (IE) is the documentation of a continuous bacteremia (>30 min in duration) based on blood culture results.

Although blood cultures remain key in making the diagnosis of IE, the need for indirect diagnostic techniques that are both specific and sensitive is increasing. This is because the nature of valvular infections has changed over the years. The numbers of fastidious organisms have increased, and the rate of the classic peripheral stigmata of IE is much lower. Patients who are elderly, chronically ill, or immunosuppressed are often afebrile and unable to mount a significant fever or exhibit the classic stigmata of valvular infection.

A major clinical challenge is that at least 25% of S aureus bloodstream infections (BSIs) represent IE or metastatic infections. The question is whether a continuous bacteremia in the presence of an intravascular line is representative of IE. Blood cultures should only be drawn through intravascular lines for the purpose of diagnosing catheter-related BSIs and have limited value for answering this clinical challenge.

Because of the ability of S aureus to produce an endotheliosis, the presence of a continuous bacteremia does not necessarily imply an infected valvular vegetation.

An important clue to continuous bacteremia/IE is the presence of S aureusbacteriuria associated with hematuria. Hematuria in the setting of IE is due to embolic renal infarction or immunologically mediated glomerulonephritis. Echocardiography has developed into a useful tool for meeting this clinical challenge.

Twenty-five percent of patients with staphylococcal bacteremia and 23% of those with catheters as the primary focus have evidence of IE based on transesophageal echocardiography (TEE) findings, in the absence of clinical and transthoracic echocardiography (TTE) findings. [2]

Blood and Urine Studies

Send baseline studies, such as complete blood count (CBC), electrolytes, creatinine, blood urea nitrogen (BUN), glucose, and coagulation panel, to the laboratory for testing.

Anemia is common in subacute endocarditis. Leukocytosis is observed in acute endocarditis. Erythrocyte sedimentation rate (ESR), while not specific, is elevated in more than 90% of cases. Decreased C3, C4, and CH50 are evident in subacute endocarditis.

Rheumatoid factor (ie, “poor man’s circulating immune complex”) becomes positive in 50% of patients with subacute disease. It becomes negative after successful treatment.

Proteinuria and microscopic hematuria are present in approximately 50% of cases.

Blood Culture

The criterion standard test for diagnosing IE is the documentation of a continuous bacteremia (>30 min in duration) based on blood culture results.

Exceptions are observed in patients with prosthetic valve endocarditis (PVE) and right-sided IE. About 5-10% of patients with IE have false-negative blood culture results. Prior use of antibiotics is the most common cause of false-negative blood culture results. Other causes include fastidious organisms and inadequate blood volume; a blood-to-broth ratio of 1:10 is needed. Currently, with modern automated blood culture systems, fastidious organisms such as nutritionally variant streptococci and members of the HACEK group rarely cause culture-negative IE.

As many as 50% of positive blood culture results have been estimated to be falsely positive. This rate has probably decreased, but false-positive blood culture results remain a major diagnostic challenge. One such result can lead to 4 days of unnecessary patient hospitalization.

The significance of positive blood culture results correlates with the following:

  • The type of organism

  • The clinical setting (coagulase-negative staphylococci [CoNS] are significant in patients with prosthetic valves but not in those with native valves)

  • Multiple blood cultures positive for the same organism

  • Shorter incubation time for recovery

  • The degree of severity of clinical illness

Procedure

Never draw only 1 set of blood cultures; 1 is worse than none. Two sets of blood cultures have greater than 90% sensitivity when bacteremia is present. Three sets of cultures improve sensitivity and may be useful when antibiotics have been administered previously.

The AHA (endorsed by IDSA) 2010 guideline update on CIED infections and their management recommends drawing at least 2 sets of blood cultures at evaluation before starting antimicrobial therapy. [42]

For diagnosing subacute IE, draw 3-5 sets of blood cultures over 24 hours. This helps detect 92-98% of cases in patients who have not recently received antibiotics. In the case of acute IE, 3 sets may be drawn over 30 minutes (with separate venipunctures) to help document a continuous bacteremia.

Using various types of blood culture bottles (with resins added to interfere with antibiotic action) probably has little advantage. Some of these may interfere with bacterial growth.

When blood culture results fail to show an infectious agent after blood is drawn 48 hours after antibiotic therapy has been stopped, the second set of blood for cultures must be drawn approximately 7 days later. If these later culture results remain negative, the diagnosis of IE must be reconsidered. In general, blood for culture should not be drawn through intravenous (IV) lines unless this is part of an approach for diagnosing line infection.

Catheter infection

The diagnosis of catheter infection may be made in 1 of 2 ways. Culturing the device via the roll-plate semiquantitative method is the most common approach but requires a catheter removal. In the case of long-term catheters, blood may be drawn simultaneously through the line and the peripheral vein. If it is impossible to draw blood from a peripheral vein in the presence of a multilumen catheter, one sample may be obtained through each of 2 catheter lumens.

In a catheter infection, the colony count of the sample obtained from the suspected port is 3-fold greater than that drawn from a peripheral vein or from another port of the catheter. Retrieval of organisms from blood drawn from a catheter hub at least 2 hours earlier before their growth is detected in the blood obtained from peripheral vein meets the differential time to positivity criteria of a catheter infection.

A sterile culture of the insertion site has a highly negative predictive value for line infection.

Culture-negative infective endocarditis

Approximately 5% of cases of possible IE yield negative blood culture results (ie, culture-negative IE). Patients with culture-negative IE occasionally present with signs and symptoms highly suggestive of IE, but the blood cultures remain negative.

Culture-negative IE may have noninfectious causes (eg, vasculitis) or may be caused by fastidious organisms. Modern blood culture systems recover the vast majority of pathogens within 4-5 days, including members of the HACEK group and Abiotrophia species. Overall, the most common cause of culture-negative IE is prior antimicrobial therapy that can suppress bacterial growth within the vegetation but is insufficient to eliminate the valvular infection.

In certain populations, infections with Coxiella burnetii (in southern France and Israel) and Bartonella species (among homeless persons) have become more frequent causes of culture-negative IE. The blood culture results in fungal valvular infections are often sterile. In S aureus IE, the blood cultures results may be negative when the organism burrows deep within the thrombus, leaving the surface of the valvular thrombus sterile (surface sterilization). [62]

Valvular vegetations may be detected during cardiac ultrasonographic examinations, but the blood culture results are persistently negative. In this situation, 3 separate blood cultures spaced over a 24-hour period are usually sufficient to detect microorganisms in the blood. Additional blood cultures are not usually helpful.

Many pathogens once considered to be fastidious are no longer classified as such (see above). Bartonella, Legionella, and C burnetii remain significant causes of culture-negative IE. These require special culture media or a prolonged incubation period for retrieval.

Serology for Chlamydia, Q fever (C burnetii), and Bartonella may be useful in culture-negative endocarditis. Serologic tests are often the most practical means for diagnosing valvular infection with fastidious organisms (eg, C burnetiiand Chlamydia, Brucella, and Legionella species). Buffered charcoal and yeast agar are required for the isolation of Legionella. Brucella species require up to 6 weeks.

It appears that Tropheryma whippelii, which causes Whipple disease, is becoming a more common cause of culture-negative endocarditis. [63]

Thuny et al have developed an algorithm for the workup of negative blood cultures. The first stage is to obtain serologies for Q fever and Bartonella. Rheumatoid factor and antinuclear antibody testing is performed to rule out rheumatologic diseases. If testing is negative, then a dedicated PCR for Bartonella species and T whipplei should be performed as well as a broad range PCR for the detection of fungi, especially in the setting of a prosthetic valve. If there is no yield from this second tier, especially with a history of antecedent antibiotic therapy, then a Septifast blood PCR should be performed to detect any staphylococci. In addition, serologic testing for Mycoplasma pneumoniae, Legionella pneumophila and Brucella melitensis should be carried out. [64]

Fungal infective endocarditis

Most types of fungal IE have a low rate of positive blood culture results. At best, only 50% of Candida species are associated with positive blood culture results. Histoplasma and Aspergillus are almost never retrieved from the bloodstream. Fungal endocarditis must always be considered in the clinical setting of culture-negative IE that fails to respond to appropriate antibiotic therapy.

Pacemaker infective endocarditis

Establishing the diagnosis of pacemaker IE is difficult because of its subtle presentation, especially late-onset disease. The addition of pocket infection and the presence of pulmonary emboli to the Duke criteria have increased the rate of diagnosis from 16% to 87.5% of cases. Fever and/or a positive blood culture result without evidence of a primary source in patients with a pacemaker or implantable cardioverter-defibrillator should be considered to represent device-associated IE until proven otherwise. [65, 66, 67, 68, 69, 70, 71]

The AHA 2010 guideline update on CIED infections recommends that, when the CIED is explanted, culture of the lead-tip and Gram stain and culture of the generator-pocket tissue be obtained. However, percutaneous aspiration of the generator pocket should not be performed for diagnostic evaluation of CIED infection. [42]

Echocardiography

Echocardiography has become the indirect diagnostic method of choice, especially in patients who present with a clinical picture of IE but who have nondiagnostic blood culture results (eg, some patients with fungal endocarditis). The diagnosis of IE can never be excluded based on negative echocardiogram findings, either from TTE or from TEE.

The AHA 2010 guideline update on CIED infections recommends TEE to evaluate the left-sided heart valves for all adults suspected of having CIED-related endocarditis, even if TTE has shown lead-adherent masses. If TTE views are good, TTE may be sufficient for pediatric patients. Patients with positive blood culture results or negative blood culture results taken after recent antimicrobial therapy should undergo TEE for CIED infection or valvular endocarditis. [42]

Visible vegetation suggests a worse prognosis. Both TTE and TEE are highly specific for valvular vegetations; however, sensitivity differs.

Two-dimensional cardiac Doppler ultrasound testing has been a significant advance for the diagnosis and evaluation of IE. It provides information about the presence and size of vegetations, which helps in diagnosis and, to some extent, in predicting embolization.

TTE is more sensitive for detecting anterior myocardial abscesses and quantitating the degree of valvular dysfunction.

The Doppler method can be used to detect distorted blood flow and certain types of cardiac pathology not otherwise visualized by standard echocardiography. It is good for visualizing jet lesions and differentiating cusp perforation from valvular insufficiency.

The combination of TEE and color Doppler is excellent for detecting intracardiac fistulas. The resolution of both TEE and TTE in real time is approximately 2 mm.

Conditions that are positively related to the detection of valvular thrombi include the location (ie, right-sided structures are poorly visualized, especially by TTE); disease lasting longer than 2 weeks; abscesses of the valves or myocardium; and aneurysms of the sinus of Valsalva.

TTE (see the image below) has generally had a sensitivity of approximately 60% for identification of valvular lesions in patients with native valve endocarditis (NVE). However, sensitivity was as high as 82% in a recent series where advanced harmonic imaging and digital processing techniques were used. [72]TTE has a sensitivity of only 20% in patients with PVE.

This is a magnified portion of a parasternal long axis view from a transthoracic echocardiogram. There is a small curvilinear vegetation on the mitral valve as indicated. The patient presented with a headache and fever, and CT scan of the brain revealed an occipital hemorrhage. The patient had a history of intravenous drug use and multiple blood cultures grew Staphylococcus aureus.

TEE was developed to overcome the problems in visualizing prosthetic valve thrombi and right-sided events. TEE eliminates the need for the operator to find a clear field for the beam. The use of higher-frequency waveforms is permitted because of the decreased distance between the heart and the probe. The sensitivity of TEE in detecting the vegetations of NVE is 90-100%. In patients with PVE, the sensitivity of TEE under optimal circumstances is greater than 90%.

TEE successfully visualizes vegetations of the leads or of the tricuspid valve in more than 90% of cases of pacemaker IE, compared with less than the 50% achieved by TTE.

Neither TEE nor TTE should be used for screening purposes (ie, patients with fever of unknown origin or those with positive blood culture results and no other signs or symptoms of IE), because nearly 60% of vegetations revealed are sterile. Approximately 15% of positive study results are false-positives because the images are, in reality, not those of vegetations but of thickened valves, nodules, or valvular calcifications.

Echocardiography is useful for predicting the potential complications of IE, especially those that are embolic in nature.

Echocardiographic predictors of systemic embolization in patients with IE are the following:

  • Large valvular vegetations (>10 mm in diameter)

  • Multiple vegetations

  • Mobile but pedunculated vegetations

  • Noncalcified vegetations

  • Vegetations that are increasing in size

  • Prolapsing vegetations

Echocardiography is also highly useful for detecting abscesses. As with valvular lesions, the transesophageal technique is generally more sensitive. [73]

In summary, the indications for performing echocardiography with Doppler in patients with IE are to provide a baseline in proven or highly suggestive cases of IE and to provide a means of documenting complications during therapy.

In most cases, TTE is sufficient. TEE is indicated when mechanical prosthetic valves are present; to detect right-sided lesions; and to visualize myocardial abscesses. Because of the endoscopic portion of the test, TEE carries the risk factor of inducing bacteremias. Approximately 15% of cases of IE do not demonstrate any detectable vegetations at the time of the echocardiographic study.

Ultrasonography

Two-dimensional cardiac Doppler ultrasound testing has been a significant advance for diagnosing and evaluating IE. It provides information about the presence and size of vegetations, which helps in diagnosis and, to some extent, in predicting embolization.

TTEs are more sensitive for detecting anterior myocardial abscesses and quantitating the degree of valvular dysfunction.

The Doppler method can detect distorted blood flow and certain types of cardiac pathology not otherwise visualized by standard echocardiography. It is good for visualizing jet lesions and differentiating cusp perforation from valvular insufficiency.

The combination of TEE and color Doppler is excellent for detecting intracardiac fistulas. The resolution of either TEE or TTE in real life is approximately 2 mm.

Conditions that are positively related to the detection of valvular thrombi are the location (ie, right-sided structures are poorly visualized, especially by TTE); disease lasting longer than 2 weeks; abscesses of the valves or myocardium; and aneurysms of the sinus of Valsalva.

Radiography

Pulmonary embolic phenomena on radiographs strongly suggest tricuspid disease (see the image below).

A young adult with a history of intravenous drug use, endocarditis involving the tricuspid valve with Staphylococcus aureus, and multiple septic pulmonary emboli. Pulmonary lesions on chest radiograph are most prominent in the right upper lobe with both solid and cavitary appearance.

Multiple embolic pyogenic abscesses may be visualized.

Other Studies

Ventilation/perfusion scanning may be useful in right-sided endocarditis.

Electrocardiography may help detect the 10% of patients who develop a conduction delay during IE by documenting an increased P-R interval. Nonspecific changes are common. First-degree atrioventricular (AV) block and new interventricular conduction delays may signal septal involvement in aortic valve disease; both are poor prognostic signs.

Catheterization of the heart is rarely required for the diagnosis of IE or any of its complications, though it may be indicated to determine the degree of valvular damage. The findings from echocardiography correlate well with the findings from cardiac catheterization. The characteristic findings of IE are intravascular endocardial vegetations that contain microorganisms surrounded by fibrin and platelets.

Various radionuclide scans using, for example, gallium (Ga)-67–tagged white cells and indium (In)-111–tagged white cells, have proven to be of little use in diagnosing IE. Radionuclide scans of the spleen are useful to help rule out a splenic abscess, which is a cause of bacteremia that is refractory to antibiotic therapy.

A computed tomography (CT) scan of the head should be obtained in patients who exhibit central nervous system (CNS) symptoms or findings consistent with a mass effect (eg, macroabscess of the brain). [74, 75, 76] This imaging modality has proven most useful for localizing abscesses. With new advanced multislice techniques, CT can now also be used to identify valvular abnormalities and vegetations. [77]

Approach Considerations

The major goals of therapy for infective endocarditis (IE) are to eradicate the infectious agent from the thrombus and to address the complications of valvular infection. The latter includes both the intracardiac and extracardiac consequences of IE. Some of the effects of IE require surgical intervention. Emergent care should focus on making the correct diagnosis and stabilizing the patient with acute disease and cardiovascular instability. General measures include the following:

  • Treatment of congestive heart failure

  • Oxygen

  • Hemodialysis (may be required in patients with renal failure)

In most cases, the etiologic microbial agent is not known while the patient is in the ED. Three sets of blood cultures should be drawn over a few hours, and then empiric antibiotic therapy tailored to the patient’s history and circumstances may be administered (see Antibiotic Therapy, below).

No special diets are recommended for patients with endocarditis; however, if the patient has congestive heart failure, administer a sodium-restricted diet. Activity limitations are determined by the severity of the illness, complications (eg, stroke), and the presence of significant congestive heart failure.

Mild congestive heart failure resulting from valvular insufficiency or myocarditis may be managed with standard medical therapy. Often, this is progressive, and despite achieving a microbiological cure, it requires valvular surgery.

Antibiotic Therapy

Antibiotics remain the mainstay of treatment for IE.

In the setting of acute IE, institute antibiotic therapy as soon as possible to minimize valvular damage. Three to 5 sets of blood cultures are obtained within 60-90 minutes, followed by the infusion of the appropriate antibiotic regimen. By necessity, the initial antibiotic choice is empiric in nature, determined by clinical history and physical examination findings.

Empiric antibiotic therapy is chosen based on the most likely infecting organisms. Native valve endocarditis (NVE) has often been treated with penicillin G and gentamicin for synergistic coverage of streptococci. Patients with a history of intravenous (IV) drug use have been treated with nafcillin and gentamicin to cover for methicillin-sensitive staphylococci. The emergence of methicillin-resistant S aureus (MRSA) and penicillin-resistant streptococci has led to a change in empiric treatment with liberal substitution of vancomycin in lieu of a penicillin antibiotic.

Prosthetic valve endocarditis (PVE) may be caused by MRSA or coagulase-negative staphylococci (CoNS) [17] ; thus, vancomycin and gentamicin may be used for treatment, despite the risk of renal insufficiency. Rifampin is necessary in treating individuals with infection of prosthetic valves or other foreign bodies because it can penetrate the biofilm of most of the pathogens that infect these devices. However, it should be administered with vancomycin or gentamicin. These latter 2 agents serve to prevent the development of resistance to the rifampin.

Substitution of linezolid for vancomycin should be considered in patients with unstable renal function because of the difficulty of achieving therapeutic trough levels in this situation.

Linezolid or daptomycin are options for patients with intolerance to vancomycin or resistant organisms. [78] Organisms with a minimum inhibitory concentration (MIC) to vancomycin of equal to or greater than 2 mcg/mL should be treated with alternative agents. Appropriate regimens should be devised in consultation with a specialist in infectious disease.

In the case of subacute IE, treatment may be safely delayed until culture and sensitivity results are available. Waiting does not increase the risk of complications in this form of the disease.

Eradicating bacteria from the fibrin-platelet thrombus is extremely difficult because (1) the high concentration of organisms present within the vegetation (ie, 10-100 billion bacteria per gram of tissue), (2) their position deep within the thrombus, (3) their location in both a reduced metabolic and reproductive state, and (4) the interference of fibrin and white cells with antibiotic action. For all of these reasons, bactericidal antibiotics are considered necessary for cure of valvular infection.

IV administration is preferred because more reliable therapeutic levels are achieved with this route. Orally administered antibiotics have been used as suppressive therapy for incurable valvular infections (ie, inoperable PVE).

Treat all patients in a hospital or skilled nursing facility to allow adequate monitoring of the development of complications and the response to antibiotic therapy.

American Heart Association guidelines for treatment

The American Heart Association (AHA) has developed guidelines for treating IE caused by the most frequently encountered microorganisms. [105]

Antibiotic doses are predicated on normal renal function.

Adult NVE caused by penicillin-susceptible S viridans, S bovis, and other streptococci (MIC of penicillin of ≤0.1 mcg/mL) should be treated with one of the following regimens:

  • Administer penicillin G at 12-18 million U/d IV by continuous pump or in 6 equally divided doses for 4 weeks

  • Administer ceftriaxone at 2 g/d IV for 4 weeks. It may be given intramuscularly (IM) for short periods if venous access problems develop; ceftriaxone allows once-a-day outpatient IV therapy for clinically stable patients.

  • Administer penicillin G and gentamicin at 1 mg/kg (based on ideal body weight) every 8 hours for 2 weeks; short-course therapy with ceftriaxone and gentamicin for 2 weeks is a cost-effective regimen and is effective in selected patients; short-course therapy is recommended for those with uncomplicated NVE caused by sensitive S viridans and of less than 3 months’ duration

  • In patients who are allergic to penicillin, use vancomycin at 30 mg/kg/d IV in 2 equally divided doses for 4 weeks; the vancomycin dose should not exceed 2 g/d unless serum levels are monitored and can be adjusted to attain a peak vancomycin level of 30-45 mcg/mL 1 hour after completion of the intravenous infusion of vancomycin

For NVE caused by relatively resistant streptococci (MICs of penicillin of 0.1-0.5 mcg/mL), the following regimens are recommended:

  • Administer penicillin G at 18 million U/d IV, either by continuous pump or in 6 equally divided doses, for 4 weeks

  • Administer cefazolin at 6 g/d IV in 3 equally divided doses for 4 weeks

  • Both of the above regimens are combined with gentamicin at 1 mg/kg (based on ideal body weight) IM or IV every 8 hours for the first 2 weeks of therapy

  • For patients who are allergic to penicillin, administer vancomycin at 30 mg/kg/d IV in 2 equally divided doses (usually, do not exceed 2 g/d unless serum levels are monitored) for 4 weeks; peak vancomycin levels of 30-45 mcg/mL should be attained 1 hour after completion of the intravenous infusion

IE caused by nonresistant enterococci, resistant S viridans (MICs of penicillin G of >0.5 mcg/mL), or nutritionally variant S viridans and PVE caused by penicillin-G–susceptible S viridans or S bovis should be treated as follows:

  • Administer penicillin G at 18-30 million U/d IV, either by continuous pump or in 6 equally divided doses daily, combined with gentamicin at 1 mg/kg (based on ideal body weight) IM or IV every 8 hours for 4-6 weeks

  • Alternatively, administer ampicillin at 12 g/d by continuous infusion or in 6 equally divided doses daily, combined with gentamicin at 1 mg/kg (based on ideal body weight) IM or IV every 8 hours for 4-6 weeks

  • In patients who are allergic to penicillin, administer vancomycin at 30 mg/kg/d in 2 equally divided doses (usually, do not exceed 2 g/24 h unless serum levels are monitored). This may be combined with gentamicin for 4-6 weeks of treatment. A peak vancomycin level of 30-45 mcg/mL should be attained 1 hour after completion of the intravenous infusion.

Enterococcal PVE generally responds as well as disease involving native valves. Six weeks of treatment is recommended for patients with symptoms of enterococcal IE of more than 3 months’ duration, with relapsed infection, or with PVE.

A combination of an inhibitor of cell wall synthesis (ie, penicillin, vancomycin) with an aminoglycoside (ie, gentamicin, streptomycin) is necessary to achieve bactericidal activity against the enterococci. Tobramycin or amikacin does not act synergistically with antibiotics active against the bacterial cell wall.

Increasing numbers of enterococci have aminoglycoside-inactivating enzymes that make them relatively resistant to the usual synergistic combinations. These aminoglycoside-resistant strains have an MIC of 2000 mcg/mL or more for streptomycin and 500 mcg/mL or more for gentamicin. Of gentamicin-resistant enterococcal strains, 25% are susceptible to streptomycin.

Continuously infused ampicillin (serum level of 16 mcg/mL) is probably the best therapy for aminoglycoside-resistant enterococci. Alternative choices are imipenem, ciprofloxacin, or ampicillin with sulbactam. Vancomycin does not appear to be as useful as the aforementioned antibiotics.

Enterococcus faecalis may become resistant to the penicillins because of their production of beta-lactamases. These strains can be treated with ampicillin combined with sulbactam or with vancomycin combined with gentamicin.

Double beta-lactam therapy (ampicillin 2 g IV every 4 hours and ceftriaxone 2 g IV every 12 hours) is recommended for treatment of enterococci susceptible to penicillin and gentamicin when the creatinine clearance is less than 50 mL/min. This combination may be effective against enterococcal isolates that are resistant to high doses of gentamicin.

High peak levels of gentamicin are not necessary to establish synergistic bactericidal activity against enterococci. Peak gentamicin levels of 3-5 mcg/mL, with a trough of less than 2 mcg/mL, frequently can be obtained with a dose of gentamicin of 1 mg/kg IV every 8 hours. Once-a-day gentamicin dosing should not be used because a prolonged postantibiotic effect against gram-positive organisms does not occur, and synergistic killing requires the simultaneous presence of an agent active in the cell wall and an aminoglycoside.

A study indicates that gentamicin usage, even for synergy, is associated with decreasing renal function. However, overall mortality does not appear to be increased. Certainly, gentamicin therapy should be continued to achieve synergy against enterococci, but the practice of administering gentamicin for 5 days in the treatment of S aureus IV drug abuse (IVDA) IE should be questioned.

Vancomycin-resistant isolates of Enterococcus faecium and Enterococcus faecalis (ie, vancomycin-resistant enterococci [VRE]) produce some of the most challenging nosocomial infections. Presently, no therapy has been proven highly effective for IE caused by strains of VRE.

Quinupristin/dalfopristin (ie, Synercid) may suppress E faecium bacteremia but frequently is not bactericidal. Other options for therapy include linezolid, a combination of ampicillin and imipenem, and chloramphenicol. In one small series, the combination of ampicillin and ceftriaxone was found to be useful against VRE. Often, the valve must be replaced to achieve a cure.

NVE caused by methicillin-sensitive S aureus (MSSA) should be treated as follows:

  • Administer nafcillin or oxacillin at 2 g IV every 4 hours for 4-6 weeks

  • Administer cefazolin at 2 g IV every 8 hours for 4-6 weeks

  • For patients who are allergic to penicillin, administer vancomycin at 30 mg/kg (usually, do not to exceed 2 g/24 h unless serum levels are monitored) for 4-6 weeks; a peak vancomycin level of 30-45 mcg/mL should be attained 1 hour after completion of the intravenous infusion.

Vancomycin therapy is associated with a significant failure rate (up to 35%) in the treatment of MSSA and MRSA bloodstream infection (BSI)/IE. It appears that vancomycin should not be used to treat infections with staphylococci with an MIC of greater than 1.5-2 mcg/mL. In these cases, alternative agents such as linezolid or daptomycin should be used.

PVE caused by MSSA should be treated as follows:

  • Administer nafcillin or oxacillin at 2 g IV every 4 hours for 6 weeks or longer

  • Alternatively, administer cefazolin at 2 g IV every 8 hours for 6 weeks or longer

  • Each of these options should be combined with rifampin at 300 mg orally every 8 hours for 6 weeks or longer and with gentamicin at 1 mg/kg (based on ideal body weight) IM or IV every 8 hours for the first 2 weeks

  • PVE caused by MRSA should be treated with vancomycin at 30 mg/kg (not to exceed 2 g/d unless serum levels are monitored) for 6 weeks or longer combined with rifampin and gentamicin as outlined above; a peak vancomycin level of 30-45 mcg/mL should be attained 1 hour after completion of the IV infusion; a significant concern is that MRSA may become resistant to vancomycin.

Treatment with linezolid appears to result in outcomes superior to those with vancomycin against many types of infections caused by MRSA and MSSA. The use of linezolid should be strongly considered instead of vancomycin in patients who are seriously ill. Another advantage of linezolid is that its dose does not need to be adjusted in patients with renal failure.

White blood cell counts, red blood cell counts, and platelet counts need to be monitored frequently while the patient is on linezolid. The risk of developing serotonin syndrome is low. After the fourth week of therapy, the risk of hematological and neuropathic complications rapidly increases.

Daptomycin (6 mg/kg/24 h) has been approved for the treatment of S aureusBSI and right-sided IE. Higher doses of daptomycin (12 mg/kg/24 h) are more effective, with little increase in adverse effects. Patients who have received vancomycin have a higher rate of resistance to daptomycin.

HACEK microorganisms should be treated as follows:

  • Administer ceftriaxone at 2 g/d IV for 4 weeks

  • Alternatively, administer ampicillin at 12 g/d by continuous pump or in 6 equally divided doses daily; this may be combined with gentamicin at 1 mg/kg (based on ideal body weight) IM or IV every 8 hours for 4 weeks

Culture-negative NVE is usually treated with vancomycin and gentamicin. In patients who have previously received antibiotics, initial therapy should consist of either ampicillin-sulbactam plus gentamicin (3 mg/kg/d) or vancomycin plus gentamicin and ciprofloxacin. Because of the increased risk of renal failure with gentamicin, the latter regimen is preferred.

Patients with culture-negative PVE are usually given vancomycin and gentamicin, targeting possible enterococcal or CoNS infections. In patients with suspected PVE who have previously received antibiotics, enteric therapy should consist of vancomycin, gentamicin, cefepime, and rifampin. Because of the risk of developing resistance to rifampin, many clinicians would start this antibiotic only after the blood cultures results become negative.

Treatment of other microorganisms is as follows:

  • For P aeruginosa, administer ceftazidime, cefepime, or imipenem, combined with high-dose tobramycin at 8 mg/kg/d in 3 divided doses, to attain peak blood levels of 15-20 mcg/mL, for 6 weeks

  • For enteric gram-negative rods (eg, E coli, Proteus mirabilis), administer ampicillin, ticarcillin-clavulanic acid, piperacillin, piperacillin-tazobactam, ceftriaxone, or cefepime combined with gentamicin or amikacin for 4-6 weeks

  • For Streptococcus pneumoniae, administer ceftriaxone at 2 g/d IV or vancomycin (if penicillin allergy or high-level penicillin G resistance [MIC of 2 mcg/mL or more]) for 4 weeks

  • For diphtheroids, administer penicillin G at 18-24 million U/d in 6 divided doses or vancomycin combined with gentamicin for 4 weeks

  • For Q fever ( C burnetii infection), administer doxycycline combined with rifampin, trimethoprim-sulfamethoxazole, or a fluoroquinolone for 3-4 years

PVE is especially difficult to treat because the microorganisms adhere to the foreign body and may make them impervious to the bactericidal action of agents active in the cell wall. All patients with PVE require at least 6 weeks of antimicrobial therapy. Rifampin is the key drug in the treatment of PVE, as it is one of the only antimicrobial agents that penetrate the biofilm laid down by S aureus and CoNS. Because of the risk of these organisms developing resistance to rifampin, many clinicians withhold the addition of rifampin until blood cultures have cleared.

Penicillin-sensitive S viridans PVE should be treated with 2 weeks of penicillin G or ceftriaxone combined with gentamicin, followed by 4 weeks of penicillin G or ceftriaxone.

If the S viridans PVE is caused by an organism with a penicillin MIC of 0.2 mcg/mL or more, penicillin G or ceftriaxone combined with gentamicin combination therapy should be administered for 4-6 weeks. If the combination therapy is administered for only 4 weeks, penicillin G or ceftriaxone should be continued for an additional 2 weeks. Vancomycin is substituted for penicillin or ceftriaxone if the patient has a history of severe, immediate penicillin hypersensitivity, such as urticaria, anaphylaxis, or angioedema.

Enterococcal PVE therapy is complicated by the multiple types of enterococcal antimicrobial resistance, including beta-lactamase production (rare), different types of aminoglycoside-inactivating enzymes (more common), and VRE (increasingly common). If the enterococci are highly resistant to both gentamicin and streptomycin, ampicillin should be administered for 8-12 weeks by continuous infusion.

No effective therapy is known for VRE PVE.

Patients with PVE must be monitored carefully for signs of valve dysfunction, congestive heart failure, and heart block. They should also be monitored for clinical response to therapy, conversion of positive blood culture results, renal function status, and serum blood levels of vancomycin and aminoglycosides.

Valve replacement surgery should be performed promptly if any of the following occurs: moderate-to-severe congestive heart failure, valve dysfunction, perivalvular or myocardial abscess formation, the presence of an unstable valve that is becoming detached from the valve ring, more than one embolic episode with persistent vegetations observed on transtracheal echocardiogram, or the presence of vegetations larger than 1 cm in diameter.

If PVE does not respond to antimicrobial therapy and blood cultures results remain positive or if a relapse of bacteremia occurs after infection, the prosthetic valve should be replaced. In the presence of microorganisms that have no microbicidal agent (eg, VRE, fungi) or in the presence of other recalcitrant organisms (eg, P aeruginosa, S aureus, enteric gram-negative rods, Brucella species, C burnetii), past clinical experience shows that early replacement of the prosthetic valve improves the chances for cure.

Fungal endocarditis is rare and primarily occurs after prosthetic valve surgery and in individuals who abuse intravenous drugs. Candida species and Aspergillus species are the organisms most frequently encountered. Currently available antifungal agents have not been successful in eliminating fungal IE. The only cures for proven fungal IE have resulted when surgical excision of the infected valves was combined with amphotericin B therapy.

Empiric therapy of IVDA IE should be aimed at S aureus. Whether to use vancomycin or oxacillin/nafcillin depends on the incidence of MRSA in the community. Generally, gram-negative organisms occur infrequently, and delay in covering them initially is acceptable.

Some clinicians obtain peak and trough blood samples during the course of antimicrobial therapy of IE in order to run serum bactericidal tests. These tests are performed by incubating serial 2-fold dilutions of serum that contains antimicrobials with an inoculum of 100,000 colony-forming units per milliliter of the target microorganism that has been previously isolated from the patient’s blood for 24-48 hours.

Peak antimicrobial concentrations that inhibit and kill the bacteria at a 1:32 or greater dilution in serum are a consistent predictor of a favorable clinical response. Antimicrobial dosages are adjusted to try to attain this goal. However, many clinicians feel that the serum bactericidal test does not have a reproducible result, and these clinicians rely on standardized tests of antimicrobial susceptibility (ie, MICs) and serum antimicrobial assays of peak and trough levels to determine whether sufficient amounts of antimicrobial agents are being administered. [79, 80, 81, 82, 83, 84]

Management of S aureus Bacteremia

In the past, management of S aureus bacteremia in the presence of an intravascular catheter required promptly removing the catheter, initiating appropriate antibiotic therapy, monitoring of blood culture results in 24-48 hours, and performing transthoracic echocardiography (TTE).

If follow-up blood culture and TTE findings were negative and no evidence of metastatic infection was found, two weeks of antistaphylococcal therapy was believed to be appropriate.

If the follow-up blood culture findings are positive, TEE should be performed. If TEE demonstrates findings of valvular infection, the patient is to be treated for 4-6 weeks with antistaphylococcal antibiotics.

Although resorting immediately to TEE is becoming more common, it is often unnecessary. A scoring system has been developed to help differentiate patients with valvular infection from those with S aureus bacteremia that represents metastatic infection from sites such as splenic abscesses or osteomyelitis. Individuals with an underlying implantable cardiac device or whose S aureus bloodstream infections developed in the community are at highest risk of IE and so should undergo immediate TEE. If the TEE findings are negative, it should be repeated in 5 days. [85]

It is important to recognize that, in up to one third of cases, a cause of persistent BSI is not identified. A good deal of these may be explained by endotheliosis. For the time being, the duration of antibiotic therapy for each case of S aureuscatheter-related BSI must be individualized. The author and others would treat cases that meet the criteria of continuous bacteremia for a total of 4 weeks despite a negative TEE result. [86, 87, 88]

Anticoagulation Therapy

Although thrombosis is a key element of IE, anticoagulation with warfarin (Coumadin, Jantoven) is controversial. Indeed, evidence indicates patients who are anticoagulated have worse outcomes than those who are not anticoagulated. Patients who are treated with anticoagulation appear to have a higher rate of intracerebral bleeding. If an established reason for anticoagulation (eg, deep venous thrombosis, presence of a mechanical prosthetic valve) exists, a standard regimen of anticoagulation should be followed.

Indications for Surgery

Approximately 15-25% of patients with IE eventually require surgery.

Indications for surgical intervention in patients with NVE are as follows:

  • Congestive heart failure refractory to standard medical therapy

  • Fungal IE (except that caused by Histoplasma capsulatum)

  • Persistent sepsis after 72 hours of appropriate antibiotic treatment

  • Recurrent septic emboli, especially after 2 weeks of antibiotic treatment

  • Rupture of an aneurysm of the sinus of Valsalva

  • Conduction disturbances caused by a septal abscess

  • Kissing infection of the anterior mitral leaflet in patients with IE of the aortic valve

Congestive heart failure in a patient with NVE is the primary indication for surgery. A second relapse, during or after completion of treatment, requires replacement of the valve.

Paravalvular abscess and intracardiac fistula almost always require surgical intervention. Patients with culture-negative NVE who remained febrile for more than 10 days should be considered surgical candidates. Persistent hypermobile vegetations, especially those with a history of embolization beyond 7 days of antibiotic therapy, should be treated with surgery. Cardiac surgery should be considered in patients with multiresistant organisms (eg, enterococci).

The indications for surgery in patients with PVE are the same as those for patients with NVE, with the addition of the conditions of valvular dehiscence and early PVE. Orally administered antibiotics have been used as suppressive therapy for incurable valvular infections (ie, inoperable PVE).

Surgery is often required for treatment of metastatic infections (eg, cerebral and other types of aneurysms and macroabscesses of the brain and spleen). Many cerebral abscesses may not be accessible. If this is the case, they can be monitored because 30% may heal when treated medically.

Occasionally, local debridement and the administration of appropriate antibiotics may be sufficient to cure an uncomplicated pacemaker pocket infection. However, most studies indicate that complete removal of the system is necessary for cure in most cases. Many patients in whom this is not possible eventually die of complications from relapsing infection. This aggressive approach is especially necessary when dealing with pacemaker IE.

The AHA 2010 guideline update on CIED infections and their management recommends complete removal of infected CIED and leads for the following patients: [42]

  • All patients with definite CIED infection, as shown by valvular and/or lead endocarditis or sepsis

  • All patients with CIED pocket infection, as shown by abscess formation, device erosion, skin adherence, or chronic draining sinus without involvement of the transvenous section of the lead system

  • All patients with valvular endocarditis without definite involvement of the lead(s),device, or both

Patients with occult staphylococcal bacteremia

The guideline update states that complete removal is reasonable in patients with persistent occult gram-negative bacteremia despite appropriate antibiotic therapy. [42]

Removal of the device and leads is not indicated in the following cases: [42]

  • A superficial or incisional infection that does not involve the device, leads, or both

  • Relapsing bloodstream infection due to a non-CIED source and for which long-term suppressive antimicrobials are required

After removal of the infected device, placing a temporary transvenous pacer is best. Immediate insertion of a permanent pacemaker at a new site can be safely accomplished.

The AHA 2010 guideline update on CIED infection recommends careful evaluation of each patient to determine if a CIED is still needed. Replacement device implantation should not be ipsilateral to the extraction site. The guideline suggests the contralateral side, the iliac vein, and epicardial implantation as preferred alternative locations. [42]

The AHA 2010 guideline recommends that if blood cultures were positive before the device extraction, blood cultures should be taken after the device removal, and new device placement should be delayed until blood cultures have been negative for at least 72 hours. If valvular infection is present, placement of new transvenous lead should be delayed for at least 14 days after CIED system removal. [42]

In the past, removal of the intracardiac leads that had been in place for several months often necessitated open heart surgery. The use of laser technology to dissolve the pacemaker lead adhesions has proven successful, with a 94% success rate. The risk of dislodging vegetations during removal of infected leads is negligible. Patients whose leads cannot be removed are started on permanent antibiotic suppression. [89, 90]

The AHA 2010 CIED guideline update states that long-term suppressive antimicrobial therapy should be considered for patients with CIED infection who are not candidates for CIED removal. Such therapy should not be administered to patients who are candidates for CIED removal. [42]

Prevention of Infective Endocarditis

Approximately 15-25% of cases of IE are a consequence of invasive procedures that produce a significant bacteremia. Because only 50% of those who developed valvular infection following a procedure were identified as being candidates for antibiotic prophylaxis, only approximately 10% of cases of IE can be prevented by the administration of preprocedure antibiotics. Maintaining good oral hygiene is probably more effective in the overall prevention of valvular infection because gingivitis is the most common source of spontaneous bacteremias.

Consider prophylaxis against IE in patients at higher risk. Patients at higher risk include those with the following conditions:

  • Presence of prosthetic heart valve

  • History of endocarditis

  • Cardiac transplant recipients who develop cardiac valvulopathy

  • Congenital heart disease with a high-pressure gradient lesion

The presence of a coronary artery stent is not considered to place the patient at high risk for endocarditis.

Also consider prophylaxis in patients before they undergo procedures [91] that may cause transient bacteremia, such as the following [92] :

  • Any procedure involving manipulation of gingival tissue or the periapical region of teeth, or perforation of the oral mucosa

  • Any procedure involving incision in the respiratory mucosa

  • Procedures on infected skin or musculoskeletal tissue including incision and drainage of an abscess

  • Prophylaxis is no longer routinely recommended for gastrointestinal or genitourinary procedures.

Prevention of vascular catheter infections is an important prophylactic approach in preventing nosocomial infective endocarditis (NIE). Protective factors include the insertion and maintenance of catheters by an infusion therapy team, the use of topical disinfectants and antibiotics, and the practice of coating catheters with antimicrobial agents.

No double-blind studies have been performed to support the use of systemically administered antibiotics for the prevention of pacemaker or intracardiac defibrillator infections. However, awaiting definitive studies, the authors recommend prophylactic antibiotics, as with any implantable device. Of course, strict sterile technique must be followed. Antibiotic prophylaxis is not recommended for prevention of CIED infection in patients with pacemakers or intracardiac defibrillators during invasive procedures not directly related to device manipulation. [42] Pacemaker infection due to transient bacteremias is uncommon. [93]

There appeared to be no dramatic increase in rates of hospitalization following the changes to the recommendations for antibiotic prophylaxis in 2007. [26]

For more information, see Antibiotic Prophylactic Regimens for Endocarditis.

American Heart Association guidelines for prophylaxis

The AHA periodically compiles recommendations for IE prophylaxis (Go to updated guidelines, October 2007). [94] It is important to remember that these are not standards but guidelines and thus may be modified in particular circumstances. The guidelines remain unproven by randomized controlled clinical trials. Indeed, many examples of failure of these recommendations have been noted, even when they are applied appropriately. [95]

The 3 major steps in the pathogenesis of IE that are vulnerable to antibiotic prophylaxis are the following:

  1. Killing of the pathogen in the bloodstream before it can adhere to the valve

  2. Preventing adherence to the valve/fibrin-platelet thrombus

  3. Eradicating any organisms that have attached to the thrombus

Successful antibiotic prophylaxis requires identifying those patients who are at risk, prioritizing the procedures that require prophylaxis, and selecting an appropriate antibiotic regimen. In general, bactericidal antibiotics are used. However, bacteristatic agents are probably effective in most circumstances.

Although the 2007 guidelines are a marked improvement because they prioritize the cardiac conditions and procedures that require antibiotic prophylaxis and emphasize the importance of promoting good oral hygiene, they offer little direction in dealing with the ever-growing problem of antibiotic-resistance patterns of S viridans and enterococci.

The importance of antibiotic prophylaxis of calcific valvular disease in elderly patients also needs to be more fully discussed. Calcific valvular disease is the most common underlying cardiac risk factor for the development of IE in this age group.

The author’s preference is to administer parenteral prophylactic antibiotics to patients with prosthetic valves because of the severe consequences of PVE.

The United Kingdom’s NICE 2008 guidelines on prophylaxis against IE differ from the AHA recommendations. The NICE guidelines do not recommend antibiotic prophylaxis for IE in patients undergoing dental procedures; however, they agree with the AHA guidelines in not recommending prophylaxis for those undergoing procedures in the upper and lower gastrointestinal tracts, the genitourinary tract, or the upper and lower respiratory tracts. [37]

Subsequent to the 2008 NICE guidelines recommending abolition of all IE antibiotic prophylaxis, prescriptions for such prophylaxis dropped almost 80% without any apparent cases of IE. [96] However, the authors of this article recommend continuing to follow the current AHA guidelines, especially in the presence of an intracardiac prosthetic device.

Special Considerations

Failure to consider the diagnosis, especially in patients with a history of IV drug use and a low-grade fever, is a medicolegal pitfall. Many malpractice suits are caused by a failure to diagnose and a delay in diagnosis accompanied by a poor outcome for the patient.

As a rule for primary care clinics, do not administer antimicrobial agents to febrile patients with heart murmurs without first obtaining at least 2 sets of blood cultures.

The perception that most IE is preventable is wrong. Frequent episodes of transient bacteremia occur with chewing and other activities of daily life. Proving that a failure to give prophylaxis before dental and surgical procedures resulted in IE is difficult. However, this does not prevent legal action alleging IE as a consequence of failing to give the antimicrobial prophylaxis recommended by the AHA.

When a central venous line is needed, not inserting the line when a patient is known to be bacteremic is advisable. If no alternative to placing the line is available, bactericidal antimicrobial agents should be administered to try to prevent the development of IE.

Consultations

In general, both a cardiologist and an infectious diseases specialist should be involved in the care of patients with IE. A bedside consultation by an infectious disease specialist results in far better outcomes than the more frequent telephone consultation. [97] Consulting a cardiothoracic surgeon may be necessary. Personnel in the clinical microbiology laboratory must have the skill to isolate the organism, properly identify it, and perform susceptibility testing appropriate for the growth characteristics and requirements of the organism (with determination of the MIC of clinically relevant antimicrobial agents). To obtain the best possible information, the attending physician should work closely with the microbiology laboratory personnel.

Long-Term Monitoring

Monitoring for posttreatment bacteremia

Patients should have blood cultures taken after 3-4 days of treatment to document eradication of the bacteremia. Blood cultures during treatment are essential if persistent fever or other signs develop that suggest failing treatment.

Failure to sterilize the bloodstream, despite adequate serum levels of appropriate antibiotics, should prompt a search for metastatic infection (eg, abscesses, especially splenic, or mycotic aneurysm).

Fever lasting longer than 10 days into therapy with an indicated antibiotic regimen should be of concern and should prompt a search for suppurative complications. Approximately 30% of patients have a return of fever after the initial response. This is usually caused by an intracardiac abscess or metastatic infection. Causes of unresponsive fever include myocardial or septal abscesses, large vegetations that resist sterilization, and metastatic infection. Occasionally, fever in patients with uncomplicated IE may take as long as 3 weeks to abate.

Monitoring for complications

Patients should be monitored for the development of the following complications:

  • Valvular dysfunction, usually insufficiency of the mitral or aortic valves

  • Myocardial or septal abscesses

  • Congestive heart failure

  • Metastatic infection

  • Embolic phenomenon

  • Organ dysfunction resulting from immunological processes

Complications, such as congestive heart failure resulting from valvular insufficiency and embolization, may occur after bacteriologic cure has been achieved. (Note that the diagnosis of developing congestive heart failure or valvular insufficiency is based on clinical findings, not solely on echocardiographic measurements.) The onset of valve dysfunction or moderate-to-severe congestive heart failure should lead to an evaluation for immediate valve replacement.

Monitoring for relapse

Relapse of IE usually occurs within 2 months of finishing clinically effective therapy. Infection with S aureus, enterococci, and gram-negative organisms (especially P aeruginosa) is associated with a high rate of relapse. Enterococcal infection of the mitral valve has the greatest potential for relapse.

Recurrent IE occurs most often in individuals who abuse IV drugs. Valvular infections in these patients recur at a rate of 40%. Those with pretreatment symptoms of IE of more than 3 months’ duration are at greater risk for relapse. Other significant risk factors for recurrence include a previous episode of IE, the presence of a prosthetic valve, and congenital heart disease.

In general, infected vascular catheters should be removed and should not be replaced over a guidewire. Surgically implanted devices, such as Broviac or Hickman catheters, do not necessarily need to be removed unless evidence of IE, a tunnel infection, or suppurative thrombophlebitis is present or if the infecting organism is a Corynebacterium species, a Pseudomonas species, a fungus, S aureus, or a Mycobacterium species. If bacteremia persists longer than a few days, the catheter must be removed. [98, 99]

Guidelines Summary

The American Heart Association issued updated guidelines for the treatment of infective endocarditis (IE) in adults in 2015. The key points are summarized below. [104]

Definition

IE is defined based on the modified Duke criteria and includes pathologic criteria and clinical criteria. The diagnosis is differentiated as definite, possible, or rejected IE.

Blood cultures

Blood cultures should be collected at least three times from different venipuncture sites. The first and second collections should be taken at least one hour apart.

Echocardiography

Transthoracic echocardiography (TTE) should be performed as quickly as possible in all cases of suspected IE. If the initial TTE images are inadequate or findings negative in the setting of persistent suspicion for IE, transesophageal echocardiography (TEE) should be performed. TEE should also be performed in patients with possible intracardiac complications in whom TTE findings were initially positive. If the suspicion for IE remains high despite negative TEE findings, a repeat TEE should be performed after 3-5 days. Furthermore, repeat TEE should be performed if a new intracardiac complication is suggested by clinical features. Performance of TTE is also reasonable upon completion of antibiotic therapy so a new baseline can be established.

Surgical intervention

The following features may merit surgical intervention:

  • Persistent vegetation following systemic embolization

  • Anterior mitral leaflet vegetation, especially larger than 10 mm

  • One or more embolic events during the first 14 days of antimicrobial therapy

  • Growing vegetation despite appropriate antimicrobial therapy

  • Acute mitral or aortic regurgitation with signs of heart failure

  • Heart failure that does not respond to medical therapy

  • Paravalvular extension

  • Valvular dehiscence, rupture, or fistula formation

  • New heart block

  • Large abscess or extension of abscess despite appropriate antimicrobial therapy

Antimicrobial therapy

The guidelines make specific recommendations for the following:

  • Native valve IE caused by highly susceptible (MIC ≤0.12 µg/mL) viridans group streptococci (VGS)

  • VGS and S bovis with MIC >0.12 µg/mL to <0.5 µg/mL

  • Abiotrophia defectiva and Granulicatella species, and VGS with penicillin MIC ≥0.5 µg/mL

  • VGS or S bovis infection of prosthetic material

  • Staphylococcal infection

  • Staphylococcal infection of prosthetic material

  • Enterococcal infection

  • Infection with HACEK micro-organisms

  • Infection with non-HACEK gram-negative bacilli

  • Culture-negative endocarditis

  • Fungal infection

Early surgery for native valve IE

The following scenarios support early valve surgery for native left-sided IE:

  • Signs or symptoms of heart failure due to valve dysfunction

  • IE due to fungal infection or highly resistant organisms

  • IE complicated by annular abscess, heart block, or destructive perforating lesions

  • Persistent infection (bacteremia or fever; >5-7 days) after appropriate antimicrobial therapy has been initiated, if other sources of fever or infection have been ruled out

  • Recurrent emboli or persistent/growing vegetations despite appropriate antimicrobial therapy

Early surgery (prosthetic valve IE)

The following scenarios support the merit consideration of early valve surgery for prosthetic valve IE:

  • Signs or symptoms of heart failure due to intracardiac fistula, valve dehiscence, or severe prosthetic dysfunction

  • Persistent bacteremia (>5-7 days) after the start of appropriate antimicrobial therapy.

  • Prosthetic valve IE complicated by annular abscess, heart block, or destructive perforating lesions

  • Prosthetic valve IE caused by fungal infection or highly resistant organisms

  • Recurrent emboli despite appropriate antimicrobial therapy

Anticoagulation

It is reasonable to discontinue all forms of anticoagulation for 2 weeks in patients with a mechanical valve and IE in whom a CNS embolic event has occurred. Antiplatelet therapy should not be initiated as adjunctive therapy upon a diagnosis of IE, although established antiplatelet therapy may be continued in patients with IE who have no bleeding complications.

CNS imaging

CNS imaging should be performed to evaluate for CNS bleeding or intracranial mycotic aneurysm in patients with IE with neurological deficits, severe localized headache, or meningeal signs.

Medication Summary

Antibiotics are the mainstay of treatment for infective endocarditis (IE). Goals to maximize treatment success are early diagnosis, accurate microorganism identification, reliable susceptibility testing, prolonged intravenous (IV) administration of bactericidal antimicrobial agents, proper monitoring of potentially toxic antimicrobial regimens, and aggressive surgical management of correctable mechanical complications.

Antibiotics

Class Summary

Empiric antimicrobial therapy must be comprehensive and should cover all likely pathogens in the context of the clinical setting.

Penicillin G (Pfizerpen)

Penicillin G is used for IE caused by S viridans or S bovis with a penicillin G minimum inhibitory concentration (MIC) of 0.1 mcg/mL or less.

Oxacillin (Bactocil in Dextrose)

Oxacillin is a bactericidal antibiotic that inhibits cell wall synthesis. It is used to treat infections caused by penicillinase-producing staphylococci. It is used to initiate therapy when MSSA infection is present.

Ceftriaxone (Rocephin)

Ceftriaxone is given as once-daily treatment of S viridans or HACEK (ie, H aphrophilus, A actinomycetemcomitans, C hominis, E corrodens, K kingae) IE. It is a third-generation cephalosporin with broad-spectrum gram-negative activity. It has lower efficacy against gram-positive organisms and higher efficacy against resistant organisms. It arrests bacterial growth by binding to one or more penicillin-binding proteins.

Vancomycin

Vancomycin is the drug of choice for patients who are allergic to penicillin who have streptococcal or enterococcal endocarditis, those with methicillin-resistant S aureus (MRSA) IE, and those with other beta-lactam–resistant gram-positive IE infections.

It is important to achieve adequate trough levels of 15-20 mcg/mL. If theMIC of the isolate is 2 mcg/mL or greater, vancomycin should not be administered.

The duration of treatment is 4 weeks in penicillin-susceptible streptococcal IE and 4-6 weeks for staphylococcal infections, prosthetic valve infections, or enterococcal IE.

Gentamicin

Gentamicin is an aminoglycoside used in combination therapy to attain bactericidal activity against enterococci and resistant streptococcal species, to shorten treatment of penicillin-susceptible streptococcal IE, and for prosthetic staphylococcal IE.

The duration of treatment is 2 weeks for penicillin-susceptible streptococcal IE and 4-6 weeks for penicillin-resistant streptococci and enterococci.

Streptomycin

Streptomycin is an aminoglycoside antibiotic that has bacteriocidal activity and acts by inhibiting protein synthesis. It may be used for the treatment of streptococcal or enterococcal endocarditis

Ampicillin

Ampicillin is used for treatment of enterococcal IE, IE caused by HACEK organisms, or as a penicillin G substitute for penicillin-susceptible organisms.

For enterococcal endocarditis, duration of treatment is 4-6 weeks in combination with gentamicin.

Ampicillin and sulbactam (Unasyn)

Ampicillin and sulbactam is a drug combination consisting of a beta-lactamase inhibitor with ampicillin. It interferes with bacterial cell wall synthesis during active replication, causing bactericidal activity against susceptible organisms.

Cefazolin

Cefazolin is a first-generation cephalosporin that is used for staphylococcal endocarditis susceptible to methicillin/oxacillin. It may be substituted for penicillin G or ampicillin for penicillin-susceptible streptococcal endocarditis. It is used if the patient develops a mild rash to penicillins but no anaphylaxis or severe immediate hypersensitivity reactions.

Ceftazidime (Fortaz, Tazicef)

Ceftazidime is a third-generation cephalosporin with broad-spectrum, gram-negative activity, including against pseudomonas. It has lower efficacy against gram-positive organisms and higher efficacy against resistant organisms. It arrests bacterial growth by binding to one or more penicillin-binding proteins, which, in turn, inhibits the final transpeptidation step of peptidoglycan synthesis in bacterial cell wall synthesis, thus inhibiting cell wall biosynthesis. The condition of the patient, severity of the infection, and susceptibility of the microorganism should determine the proper dose and route of administration.

Cefepime (Maxipime)

Cefepime is a fourth-generation cephalosporin with gram-negative coverage comparable to ceftazidime, but it has better gram-positive coverage (comparable to ceftriaxone). Cefepime is a zwitter ion and rapidly penetrates gram-negative cells.

Nafcillin (Nallpen in Dextrose)

Nafcillin is used for staphylococcal IE caused by organisms susceptible to methicillin/oxacillin.

Linezolid (Zyvox)

Linezolid prevents formation of functional 70S initiation complex, which is essential for bacterial translation process. It is bacteriostatic against enterococci and staphylococci and bactericidal against most strains of streptococci. It is used as an alternative in patients allergic to vancomycin and for treatment of vancomycin-resistant enterococci (VRE).

The FDA warns against the concurrent use of linezolid with serotonergic psychiatric drugs, unless indicated for life-threatening or urgent conditions. Linezolid may increase serotonin CNS levels as a result of MAO-A inhibition, increasing the risk of serotonin syndrome.

Daptomycin (Cubicin)

Daptomycin is a cyclic lipopeptide antibiotic that is used for Staphylococcus aureus bacteremia, including those with right-sided infective endocarditis caused by MSSA or MRSA. There is a risk of developing eosinophilic pneumonia with daptomycin use. Immediate discontinuation of daptomycin is recommended if eosinophilic pneumonia is suspected.

Rifampin (Rifadin)

Rifampin is used synergistically in the treatment of staphylococcal infections associated with a foreign body, such as a prosthetic heart valve.

Rifampin inhibits DNA-dependent RNA polymerase activity in susceptible cells. Specifically, it interacts with bacterial RNA polymerase but does not inhibit the mammalian enzyme. Cross-resistance has been shown only with other rifamycins.

Ciprofloxacin (Cipro, Cipro XR)

Ciprofloxacin is a fluoroquinolone antibiotic that inhibits bacterial DNA synthesis and, consequently, growth, by inhibiting DNA gyrase and topoisomerases, which are required for replication, transcription, and translation of genetic material. Quinolones have broad-spectrum activity against gram-positive and gram-negative aerobic organisms. It has no activity against anaerobes.

Doxycycline (Adoxa, Doryx, Monodox, Vibramycin)

Doxycycline is a broad-spectrum, synthetically derived bacteriostatic antibiotic in the tetracycline class. It is almost completely absorbed, concentrates in bile, and is excreted in urine and feces as a biologically active metabolite in high concentrations. It inhibits protein synthesis and, thus, bacterial growth by binding to the 30S and possibly 50S ribosomal subunits of susceptible bacteria. It may block dissociation of peptidyl t-RNA from ribosomes, causing RNA-dependent protein synthesis to arrest.

Sulfamethoxazole and Trimethoprim (Bactrim, Bactrim DS, Septra DS, Sulfatrim)

Trimethoprim and sulfamethoxazole inhibit bacterial growth by inhibiting the synthesis of dihydrofolic acid.

Quinupristin and dalfopristin (Synercid)

Quinupristin and dalfopristin belong to the macrolide/lincosamide/streptogramin group of antibiotics. The combination inhibits protein synthesis and is usually bacteriostatic. This drug combination is effective against Enterococcus faecium, but not Enterococcus faecalis strains. It is an option for the treatment of vancomycin-resistant E faecium infections.

Ticarcillin-clavulanate (Timentin)

Ticarcillin-clavulanate inhibits the biosynthesis of cell wall mucopeptide and is effective during the stage of active growth. The drug combination includes an antipseudomonal penicillin plus a beta-lactamase inhibitor that provides coverage against most gram-positive, most gram-negative, and most anaerobic organisms.

Piperacillin and Tazobactam sodium (Zosyn)

combination includes an antipseudomonal penicillin plus beta-lactamase inhibitor. It inhibits biosynthesis of cell wall mucopeptide and is effective during the stage of active multiplication.

Entradas destacadas
Entradas recientes
Archivo
Buscar por tags
Síguenos
  • Facebook Basic Square
  • Twitter Basic Square
  • Google+ Basic Square
bottom of page